Крок 2 - Медицина 2020 1 день (буклет)

1 / 130
Жінка 42-х років прийшла до лікаря на профілактичний прийом. Скарг не має. Загальний стан задовільний. Зріст 162 см, маса тіла 87 кг, ІМТ = 33 кг/м2. Артеріальний тиск справа - 140/90 мм рт.ст., зліва - 145/85 мм рт.ст., пульс - 72/хв. Аускультативно тони серця приглушені, шуми не прослуховуються. В легенях дихання везикулярне. Нижній край печінки виступає на 1,5-2 см з-під нижнього краю. Набряків не виявлено. Яку оцінку конституціональній будові тіла цієї пацієнтки має дати лікар (за рекомендаціями ВООЗ)? A 42-year-old woman came to the doctor for a preventive appointment. She has no complaints. The general condition is satisfactory. Height 162 cm, body weight 87 kg, BMI = 33 kg /m2. Arterial pressure on the right - 140/90 mm Hg, on the left - 145/85 mm Hg, pulse - 72/min. On auscultation, heart sounds are muffled, no murmurs can be heard. Breathing is vesicular in the lungs. The lower edge of the liver protrudes 1.5-2 cm from the lower edge. No edema was detected. What assessment of the constitutional structure of this patient's body should the doctor give (according to WHO recommendations)?

Ожиріння І ступеня Obesity of the 1st degree

Ожиріння III ступеня Obesity III degree

Надлишкова вага Overweight

Нормальна вага Normal weight

Ожиріння II ступеня Obesity II degree

2 / 130
Жінка 35-ти років звернулася до лікаря зі скаргами на зростаючий біль під час менструації протягом 1-го року. Тривалість менструального циклу 28 днів. 2 роки тому була проведена лапароско- пічна перев’язка маткових труб. При пальпації матка збільшена до 8 тижнів вагітності, м’якої консистенції, болюча. Придатки з обох сторін не збільшені, при пальпації безболісні. Який діагноз є найбільш імовірним? A 35-year-old woman consulted a doctor with complaints of increasing pain during menstruation during the 1st year. The length of the menstrual cycle is 28 days. 2 years ago she was laparoscopic tubal ligation. Upon palpation, the uterus is enlarged to 8 weeks of pregnancy, soft in consistency, painful. The appendages on both sides are not enlarged, and are painless upon palpation. What diagnosis is most likely?

Полікістоз яєчників Polycystic ovary

Аденоміоз Adenomyosis

Гідросальпінкс Hydrosalpinx

Зовнішній ендометріоз External endometriosis

Позаматкова вагітність Ectopic pregnancy

3 / 130
Чоловік 32-х років звернувся до лікаря зі скаргами на висип, що з’явився на колінах 4 місяці тому і зараз вперше починає з’являтися на ліктях. Зі слів пацієнта болю не відчуває, але місце висипу злегка свербить та кровоточить при розчухуванні. Він відзначив, що на ліктях висип з’явився після подряпин, які йому наніс його домашній кіт. При фізикально- му обстеженні лікарем виявлено на колінах та ліктях еритематозні бляшки діаметром від 2 до 3 см із чітким контуром, щільні, вкриті сріблястими лусочками. Який діагноз є найбільш імовірним? A 32-year-old man went to the doctor complaining of a rash that appeared on his knees 4 months ago and is now beginning to appear on his elbows for the first time. With according to the patient's words, he does not feel pain, but the rash is slightly itchy and bleeds when scratched. He noted that the rash appeared on his elbows after being scratched by his pet cat. During a physical examination, the doctor revealed erythematous plaques with a diameter of from 2 to 3 cm with a clear outline, dense, covered with silvery scales. What is the most likely diagnosis?

Себорейний дерматит Seborrheic dermatitis

Псоріаз Psoriasis

Контактний дерматит Contact dermatitis

Екзема Eczema

Хвороба котячих подряпин Cat Scratch Disease

4 / 130
Мати хлопчика 6-ти місяців прийшла на прийом до педіатра стурбована тим, що її дитина не отримувала жодних щеплень. Скарг не мають. Об’єктивно: температура - 37,1°С, частота дихання - 20/хв., артеріальний тиск - 100/70 мм рт.ст. Зріст та вага відповідають діапазону між 50 та 75 перцентилем. Відповідно до нормативно-правових документів МОЗ, якої тактики має дотримуватися лікар щодо імунізації цієї дитини? The mother of a 6-month-old boy came to see a pediatrician worried that her child had not received any vaccinations. They have no complaints. Objectively: temperature - 37 ,1°C, respiratory rate - 20/min, blood pressure - 100/70 mm Hg. Height and weight correspond to the range between the 50th and 75th percentile. According to the regulatory and legal documents of the Ministry of Health, what tactics should a doctor follow regarding immunization of this child?

Ввести вакцину БЦЖ, КПК та АКДС Inject BCG, CPC and DPT vaccine

Ввести вакцину БЦЖ Enter BCG vaccine

- -

Провести пробу Манту та записати на прийом через 2-3 дні Conduct a Mantoux test and make an appointment in 2-3 days

Відкласти вакцинацію та записати на прийом через тиждень Postpone the vaccination and make an appointment in a week

5 / 130
Хвора 64-х років скаржиться на загальну слабкість, шум у голові, оси- плість голосу. Об’єктивно: блідість з жовтяничним відтінком, язик червоний зі згладженими сосочками, асиметрія тактильної і больової чутливості. Пульс - 120/хв., артеріальний тиск - 80/50 мм рт.ст. Пальпується селезінка. У крові: НЬ- 58 г/л, еритроцити - 1,2- 1012/л, лейкоцити - 2,8-109/л, тромбоцити - 140- 109/л, швидкість осідання еритроцитів - 17 мм/год, анізоцитоз, пойкі- лоцитоз - виражені (++). Яке дослідження буде вирішальним у з’ясуванні генезу анемії? A 64-year-old patient complains of general weakness, noise in the head, hoarseness of voice. Objectively: paleness with a jaundiced tint, red tongue with smoothed papillae , asymmetry of tactile and pain sensitivity. Pulse - 120/min., blood pressure - 80/50 mm Hg. The spleen is palpated. In the blood: Hb - 58 g/l, erythrocytes - 1.2 - 1012/l, leukocytes - 2.8-109/l, platelets - 140-109/l, erythrocyte sedimentation rate - 17 mm/h, anisocytosis, poikilocytosis - pronounced (++). Which study will be decisive in clarifying the genesis of anemia?

Стернальна пункція Sternal puncture

Пряма проба Кумбса Direct Coombs test

Непряма проба Кумбса Indirect Coombs Test

Люмбальна пункція Lumbar puncture

Фіброгастроскопія Fibrogastroscopy

6 / 130
Вам довелося прийти на допомогу жінці 58-ми років, у якої виникла рясна кровотеча з розірваного варикозно розширеного венозного вузла на лівій гомілці. У чому полягає перша допомога? You had to attend to a 58-year-old woman who was bleeding profusely from a ruptured varicose vein in her left leg. What is first aid?

Операція Троянова-Тренделенбурга Troyanov-Trendelenburg operation

Накладання джгута дистально до джерела кровотечі Applying a tourniquet distal to the source of bleeding

Накладання джгута проксимально до джерела кровотечі Applying a tourniquet proximal to the source of bleeding

Підвищене положення кінцівки. Стискальна стерильна пов’язка Elevated limb position. Compression sterile bandage

Z-подібний шов на розірваний варикозний вузол Z-shaped suture on a ruptured varicose node

7 / 130
Хвора 36-ти років скаржиться на задишку, відчуття стискання в правій половині грудної клітки, підвищення температури до 38,1°С, кашель з виділенням невеликої кількості слизово-гнійного харкотиння. Хворіє понад тиждень. Скарги пов’язує з переохолодженням. Об’єктивно: легкий акроціаноз губ, пульс ритмічний, 90/хв., артеріальний тиск - 140/85 мм рт.ст. Права половина грудної клітки відстає в акті дихання. Перкусія - справа нижче кута лопатки прослухо- вується тупість з межею до верху В цій ділянці дихання відсутнє. Який найбільш імовірний діагноз? A 36-year-old patient complains of shortness of breath, a feeling of tightness in the right half of the chest, an increase in temperature to 38.1°C, a cough with the release of a small amount of mucous purulent sputum. He has been ill for more than a week. He relates the complaints to hypothermia. Objectively: slight acrocyanosis of the lips, rhythmic pulse, 90/min., blood pressure - 140/85 mm Hg. The right half of the chest lags behind in the act of breathing. Percussion - on the right below the angle of the scapula, dullness is heard with a border towards the top. There is no breathing in this area. What is the most likely diagnosis?

Абсцес легені Lung abscess

Позагоспітальна пневмонія Community-acquired pneumonia

Ателектаз легені Atelectasis of the lung

Ексудативний плеврит Exudative pleurisy

ТЕЛА BODIES

8 / 130
Хворий 20-ти років пройшов курс по- ліхіміотерапії за схемою ВАМП з приводу гострого лімфо бластного лейкозу. Яка морфологічна картина кісткового мозку може свідчити про настання ремісії? A 20-year-old patient underwent a course of polychemotherapy according to the VAMP scheme for acute lymphoblastic leukemia. What morphological picture of the bone marrow can indicate the onset of remission?

Вміст бластних клітин до 15% Content of blast cells up to 15%

Вміст бластних клітин до 1 % Content of blast cells up to 1%

Вміст бластних клітин до 5% Content of blast cells up to 5%

Відсутність бластних клітин No blast cells

Вміст бластних клітин до 10% Content of blast cells up to 10%

9 / 130
Жінка 36-ти років скаржиться на біль, обмеження рухів у дрібних суглобах рук, ускладнення при ковтанні твердої їжі, слабкість, сухий кашель. Об’єктивно: шкіра кистей та передпліччя щільна, гладенька. Проксимальні суглоби II-IV пальців кистей рук набряклі, болючі при пальпації. Над легенями сухі розсіяні хрипи, межі серця зміщені вліво на 2 см, тони приглушені. У крові: швидкість осідання еритроцитів - 36 мм/год, 7-глобулінів - 24%. У сечі: змін немає. Який найбільш імовірний діагноз? A 36-year-old woman complains of pain, limitation of movement in the small joints of the hands, difficulty swallowing solid food, weakness, dry cough. Objectively: the skin of the hands and the forearm is dense, smooth. The proximal joints of the II-IV fingers of the hands are swollen, painful on palpation. Over the lungs, dry scattered rales, the borders of the heart are shifted to the left by 2 cm, the sounds are muffled. In the blood: the sedimentation rate of erythrocytes - 36 mm/h, 7 -globulins - 24%. In the urine: there are no changes. What is the most likely diagnosis?

Дерматоміозит Dermatomyositis

Ревматоїдний артрит Rheumatoid arthritis

Системна склеродермія Systemic scleroderma

Саркоїдоз Sarcoidosis

Системний червоний вовчак Systemic lupus erythematosus

10 / 130
У хворого 35-ти років, що страждає на хронічний гломерулонефрит і останні 3 роки перебуває на гемодіалізі, з’явилися перебої в роботі серця, гіпотонія, зростаюча слабкість, задишка. На ЕКГ: брадикардія, атріовентрикулярна блокада І ст., високі загострені зубці Т. Напередодні - грубе порушення питного та дієтичного режимів. Яка найбільш імовірна причина вказаних змін? A 35-year-old patient who suffers from chronic glomerulonephritis and has been on hemodialysis for the past 3 years has had heart failure, hypotension, increasing weakness, shortness of breath. On the ECG: bradycardia, atrioventricular blockade of the 1st degree, high pointed T waves. The day before - a gross violation of the drinking and dietary regimes. What is the most likely cause of the indicated changes?

Гіпокальціємія Hypocalcemia

Гіперкаліємія Hyperkalemia

Гіпернатріємія Hypernatremia

Гіпокаліємія Hypokalemia

Гіпергідратація Hyperhydration

11 / 130
Хворий 30-ти років працює клепальником протягом 6-ти років. Скарги на різкий біль у плечовому поясі, особливо в нічний час. Пальці рук німіють та біліють при охолодженні. Став дратівливим. Об’єктивно: кисті набряклі, холодні, ціанотичні. Різко знижені всі види чутливості (больова, температурна, вібраційна). При пробі на холод - симптом ”мертвих пальців”. Відзначається слабкість у привідних м’язах V пальця; змінена електрозбудженість. Сухожилкові та періостальні рефлекси живі. Який попередній діагноз? A 30-year-old patient has been working as a riveter for 6 years. Complains of sharp pain in the shoulder girdle, especially at night. The fingers are numb and turn white when cooled . Became irritable. Objectively: the hands are swollen, cold, cyanotic. Sharply reduced all types of sensitivity (pain, temperature, vibration). When tested for cold - the symptom of 'dead fingers'. There is weakness in the adductor muscles of the V finger; changed electrical excitability. Tendon and periosteal reflexes are alive. What is the previous diagnosis?

Вібраційна хвороба загальної дії Vibration disease of general effect

Синдром Рейно Raynaud's syndrome

Вібраційна хвороба локальної дії Vibration disease of local effect

Вузликовий періартерїїт Nodular periarteritis

Вегетативно-сенсорна полінейропатія Vegetative-sensory polyneuropathy

12 / 130
При проведенні медичного огляду учнів середнього та старшого шкільного віку лікарі визначали відповідність біологічного розвитку та календарного віку за наступними критеріями: щорічне збільшення довжини тіла, осифікація кісток кисті, кількість постійних зубів. Який додатковий показник розвитку у ці вікові періоди найбільш імовірно мають включити лікарі? When conducting a medical examination of middle and high school students, doctors determined the correspondence of biological development and calendar age according to the following criteria: annual increase in body length, ossification of hand bones, number of permanent teeth. What additional developmental indicator should doctors most likely include in these age periods?

Розвиток вторинних статевих ознак Development of secondary sexual characteristics

Маса тіла Body weight

Обвід грудної клітки Chest Circumference

Життєва ємність легень Vital lung capacity

М’язова сила кисті Hand muscle strength

13 / 130
Хлопчика 2-х років госпіталізовано з приводу зменшення маси тіла, нестійких випорожнень, анорексії, які з’явилися після введення в раціон манної каші (з 5 місяців). Дитина адинамічна, млява, шкіра бліда, суха, підшкірно-жировий шар відсутній. Живіт здутий, напружений. Під час перкусії у верхній частині живота тимпаніт, шум плеску, випорожнення пінисті, світлого кольору, смердючі. В копроцитограмі: нейтральний жир - багато. Який наступний крок у веденні пацієнта буде найбільш доречним? A 2-year-old boy was hospitalized due to a decrease in body weight, unstable stools, anorexia, which appeared after the introduction of semolina into the diet (from 5 months). The child is adynamic, lethargic, the skin is pale, dry, the subcutaneous fat layer is absent. The abdomen is distended, tense. During percussion in the upper part of the abdomen, tympanitis, splashing noise, stools are foamy, light-colored, smelly. In the coprocytogram: neutral fat - a lot. What What would be the most appropriate next step in patient management?

Рентгенографія органів черевної порожнини X-ray of abdominal organs

Розробка індивідуального плану харчування з великим вмістом клітковини Development of an individual meal plan with high fiber content

Призначення антибіотиків широкого спектру дії Prescription of broad-spectrum antibiotics

Негайне оперативне втручання Immediate operative intervention

Визначення IgA до тканинної транс- глютамінази Determination of IgA to tissue transglutaminase

14 / 130
При вивченні середнього рівня та характеру різноманітності деяких лабораторних показників отримані такі дані: для загального білку крові - середнє квадратичне відхилення ±4 г/л, коефіцієнт варіації - 6%; для швидкості осідання еритроцитів, відповідно, ±2 мм/год, 23%. Яка з ознак, що вивчаються, є найбільш різноманітною? When studying the average level and nature of the diversity of some laboratory indicators, the following data were obtained: for total blood protein - mean square deviation ±4 g/l, coefficient of variation - 6% ; for the sedimentation rate of erythrocytes, respectively, ±2 mm/h, 23%. Which of the studied features is the most diverse?

Для вивчення різноманітності потрібні додаткові розрахунки More calculations are required to study diversity

Швидкість осідання еритроцитів (ШОЕ) Erythrocyte Sedimentation Rate (ESR)

Для вивчення різноманітності потрібні додаткові дослідження More research is needed to study diversity

Загальний білок сироватки крові Total serum protein

Відмінності в різноманітності ознак відсутні There are no differences in the variety of signs

15 / 130
Хворий 48-ми років госпіталізований з нападами судом. Багато років страждає на виразку 12-палої кишки (ДПК). Протягом останнього місяця спостерігає щоденне блювання, схуд на 20 кг. Об’єктивно: хворий виснажений. В епі- гастральній ділянці визначається ”шум плеску”. Нижня межа шлунка на рівні гребінцевої лінії. Лабораторно: загальний білок - 47 г/л; К - 2,1 ммоль/л, Na - 118 ммоль/л, Са -1,6 ммоль/л, хлориди - 82 ммоль/л, гематокрит - 64%. Який попередній діагноз? A 48-year-old patient is hospitalized with seizures. He has been suffering from a duodenal ulcer for many years. During the last month, he has been vomiting daily, he has lost weight by 20 kg. Objectively: the patient is exhausted. A 'slap noise' is detected in the epigastric region. The lower border of the stomach is at the level of the comb line. Laboratory: total protein - 47 g/l; K - 2.1 mmol/l, Na - 118 mmol/l, Ca -1.6 mmol/l, chlorides - 82 mmol/l, hematocrit - 64%. What is the previous diagnosis?

Компенсований стеноз воротаря Compensated stenosis of the goalkeeper

Субкомпенсований стеноз воротаря Subcompensated stenosis of the goalkeeper

Малігнізована виразка шлунка Malignant gastric ulcer

Кровоточива виразка 12-палої кишки Bleeding duodenal ulcer

Декомпенсований стеноз воротаря Decompensated gatekeeper stenosis

16 / 130
Хворий 39-ти років скаржиться на задишку та стискаючий біль за грудиною у стані спокою. 10 днів тому переніс грип. Об’єктивно: поза вимушена - сидить, нахиливши тулуб уперед, обличчя одутле, ціанотичне, здуті шийні вени. Межі серця значно розширені в обидві сторони, тони глухі, ритмічні, частота серцевих скорочень - 104/хв., частота дихальних рухів - 28/хв. На ЕКГ: зниження вольтажу зубців, конкордантні зміни сегменту ST. На рентгенограмі: кулеподібна тінь серця, ознаки застою в легенях. В крові: швидкість осідання еритроцитів - 38 мм/год. Поставте діагноз: A 39-year-old patient complains of shortness of breath and squeezing pain behind the sternum at rest. He had the flu 10 days ago. Objectively: the position is forced - he sits, leaning the body is forward, the face is swollen, cyanotic, distended neck veins. Heart borders are significantly expanded on both sides, tones are dull, rhythmic, heart rate - 104/min, respiratory rate - 28/min. On the ECG: a decrease in the voltage of the teeth, concordant changes in the ST segment. On the radiograph: a spherical shadow of the heart, signs of congestion in the lungs. In the blood: the sedimentation rate of erythrocytes is 38 mm/h. Make a diagnosis:

Ексудативний перикардит Exudative pericarditis

Дилатаційна кардіоміопатія Dilated cardiomyopathy

Ревматична гарячка Rheumatic fever

Нестабільна стенокардія Unstable angina

Вірусний міокардит Viral myocarditis

17 / 130
Дівчинка 6-ти років надійшла зі скаргами на підвищення температури тіла до 37,2°С', часті та болісні сечовипускання, які з’явилися після переохолодження. У сечі: сеча каламутна, питома вага -1012, білок - 0,033°/оо, мікроскопія: лейкоцити - 40-45 в п/з, еритроцити - 8-9 в п/з (свіжі), епітелій плаский: 5-8 в п/з, слиз. Який етіологічний фактор у даному випадку найбільш імовірний? A 6-year-old girl came in with complaints of an increase in body temperature to 37.2°С', frequent and painful urination, which appeared after hypothermia. In urine: cloudy urine, specific gravity -1012, protein - 0.033°/oo, microscopy: leukocytes - 40-45 in p/z, erythrocytes - 8-9 in p/z (fresh), squamous epithelium: 5-8 in p /z, mucus. What is the most probable etiological factor in this case?

Klebsiella pneumoniae Klebsiella pneumoniae

Candida albicans Candida albicans

Staphylococcus aureus Staphylococcus aureus

Proteus mirabilis Proteus mirabilis

Escherichia coli Escherichia coli

18 / 130
Хвора 50-ти років, яка страждає на хворобу Аддісона (виникла після перенесеного туберкульозу легень у юному віці), під час пожежі в квартирі сильно налякалася і знепритомніла. Хвора бліда, холодна, пульс ниткоподібний, частота серцевих скорочень - 120/хв., артеріальний тиск - 60/30 мм рт.ст. Яке ускладнення виникло у хворої? A 50-year-old patient suffering from Addison's disease (occurred after suffering from pulmonary tuberculosis at a young age), was very frightened and fainted during a fire in her apartment. The patient pale, cold, thread-like pulse, heart rate - 120/min., blood pressure - 60/30 mm Hg. What complication did the patient have?

Тиреотоксичний криз Thyrotoxic crisis

Гостра наднирникова недостатність Acute adrenal insufficiency

Тампонада серця Cardiac tamponade

Синдром Морганьї-Адамса-Стокса Morganhi-Adams-Stokes syndrome

Гострий інфаркт міокарда Acute myocardial infarction

19 / 130
Чоловік 45-ти років доставлений у відділення невідкладної допомоги зі скаргами на раптовий інтенсивний біль у поперековій ділянці, часте болісне сечовиділення, блювання. Температура тіла - 36,8°С*, пульс - 82/хв., артеріальний тиск - 130/80 мм рт.ст. При фізикальному обстеженні болісність при пальпації поперекової ділянки відсутня, позитивний симптом Пастернацького. При лабораторному дослідженні у загальному ана-лізі сечі білок - 0,06 г/л, реакція - слабко кисла, лейкоцити - 3-4 в полі зору, еритроцити - 30-40 в полі зору. Який діагноз є найбільш імовірним? A 45-year-old man was brought to the emergency department with complaints of sudden intense pain in the lumbar region, frequent painful urination, vomiting. Body temperature - 36.8° C*, pulse - 82/min., arterial pressure - 130/80 mm Hg. During physical examination, there is no pain during palpation of the lumbar region, positive Pasternacki's symptom. During laboratory research, in the general analysis of urine, protein - 0.06 g /l, the reaction is weakly acidic, leukocytes - 3-4 in the field of vision, erythrocytes - 30-40 in the field of vision. What diagnosis is the most likely?

Гострий пієлонефрит Acute pyelonephritis

Сечокам’яна хвороба Urolithiasis

Полікістоз нирок Polycystic kidneys

Гостра ниркова недостатність Acute renal failure

Гострий гломерулонефрит Acute glomerulonephritis

20 / 130
Хворий 34-х років перебуває на лікуванні в психіатричній лікарні з приводу загострення шизофренії. Об’єктивно: перебуває в ліжку, рухливо загальмований, контакт відсутній. На запитання не відповідає. Поза одноманітна, пацієнт гіпомімічний, наявні симптом ”хоботка”, воскова гнучкість м’язів, симптом ”повітряної подушки”. В такому стані лишається близько тижня. Харчування парентеральне. Визначте наявний синдром розладу рухово-вольової сфери: A 34-year-old patient is being treated in a psychiatric hospital due to an exacerbation of schizophrenia. Objectively: he is in bed, physically inhibited, there is no contact. When asked, no responds. The posture is monotonous, the patient is hypomimic, the 'proboscis' symptom, waxy flexibility of the muscles, the 'air cushion' symptom are present. He remains in this state for about a week. Parenteral nutrition. Define the present syndrome of motor-volitional disorder:

Екзогенний ступор Exogenous stupor

Депресивний ступор Depressive stupor

Психогенний ступор Psychogenic stupor

Апатичний ступор Apathetic stupor

Кататонічний ступор Catatonic stupor

21 / 130
У хворого 47-ми років на восьмий день після операції з приводу панкреоне- крозу, перитоніту з’явилось підвищення температури тіла до 39°С*, анемія, лейкоцитоз, гіпопротеїнемія, спленомега- лія, токсична енцефалопатія. Яке ускладнення має розвиток у цій ситуації? In a 47-year-old patient, on the eighth day after surgery for pancreonecrosis, peritonitis, an increase in body temperature up to 39°С*, anemia, leukocytosis , hypoproteinemia, splenomegaly, toxic encephalopathy. What complications does development have in this situation?

Гостра ниркова недостатність Acute renal failure

Гостра печінкова недостатність Acute liver failure

Внутрішньоочеревинна кровотеча Intraperitoneal bleeding

Загальна гнійна інфекція (сепсис) General purulent infection (sepsis)

Гострий менінгіт Acute meningitis

22 / 130
На 8-му добу життя у новонародженого хлопчика з масою тіла 3500 г підвищується температура тіла та з’являється висип. Напередодні дитина була дуже дратівлива та мала субфебрильну температуру, що поступово підвищувалася. Мати повідомила, що до зазначе-ного епізоду дитина була здорова та ніяких ліків не приймала. Температура тіла - 38,9°С*, артеріальний тиск - 90/50 мм рт.ст., пульс - 460/хв., частота дихання - 47/хв., SpO2 - 98% при кімнатному повітрі. При огляді виявлено висип у вигляді пухирів, що легко лопаються. Висип охоплює 60% поверхні тіла, є навкруги рота, але не виявлено на слизових оболонках. Позитивний симптом Ніколь- ського. Який діагноз є найбільш імовір-ним? On the 8th day of life, a newborn boy with a body weight of 3500 g has a fever and a rash appears. The day before, the child was very irritable and had a low-grade fever, which gradually increased. The mother reported that the child was healthy before the mentioned episode and did not take any medications. Body temperature - 38.9°С*, blood pressure - 90/50 mm Hg, pulse - 460/min. , respiratory rate - 47/min, SpO2 - 98% on room air. On examination, a rash in the form of blisters that burst easily. The rash covers 60% of the body surface, is around the mouth, but not on the mucous membranes. Positive symptom Nicole What is the most likely diagnosis?

Стафілококовий синдром ”обпеченої шкіри” Staphylococcal 'burnt skin' syndrome

Токсичний епідермальний некроліз Toxic epidermal necrolysis

Синдром Стівенса-Джонсона Stevens-Johnson Syndrome

Вроджена вітряна віспа Congenital chicken pox

Кропив’янка Hives

23 / 130
У повторнороділлі 30-ти років пологи тривають 8 годин. Перейми через кожну хвилину по 50 секунд, активні. Серцебиття плоду - Г56/хв., ритмічне. Під час зовнішнього дослідження голівка розташована в порожнині малого тазу. Вагінально: розкриття шийки матки повне, голівка плоду в площині виходу з малого тазу. Стрілоподібний шов в пря-мому розмірі, мале тім’ячко біля лона. Який це період пологів? In a repeat birth of 30 years, labor lasts 8 hours. Contractions every minute for 50 seconds, active. Fetal heartbeat - G56/min., rhythmic. During external examination, the head is located in the cavity of the small pelvis. Vaginal: the opening of the cervix is complete, the fetal head is in the plane of the exit from the small pelvis. The arrow-shaped seam is in direct size, the small crown near the womb. What is the period of childbirth?

Латентна фаза першого періоду нормальних пологів Latent phase of the first period of normal childbirth

Другий період нормальних пологів Second period of normal childbirth

Стрімкі пологи Rapid childbirth

Прелімінарний період Preliminary period

Активна фаза першого періоду нормальних пологів Active phase of the first period of normal childbirth

24 / 130
Хворий скаржиться на гнійні виділення з пенісу вранці перед сечовиділенням, різі, полакіурію. Визначена піурія при проведені трьохстаканної проби в першій порції сечі. Поставте діагноз: The patient complains of purulent discharge from the penis in the morning before urinating, cuts, pollakiuria. Pyuria was determined when a three-glass sample was taken in the first portion of urine. Make a diagnosis:

Хронічний пієлонефрит Chronic pyelonephritis

Хронічний простатит Chronic prostatitis

Гострий цистит Acute cystitis

Гострий уретрит Acute urethritis

Гострий баланопостит Acute balanoposthitis

25 / 130
Дитина 6-ти років з наявністю анемічного, геморагічного синдромів. В крові: НЬ- 80 г/л, КП- 0,9, ретикулоцити - 2°/оо, лейкоцити - 1,0- 109/л, тромбоцити - 10- 109/л. Який діагноз є найбільш імовірним? A 6-year-old child with anemic, hemorrhagic syndromes. In the blood: Hb - 80 g/l, CP - 0.9, reticulocytes - 2°/ oo, leukocytes - 1.0-109/l, platelets - 10-109/l. What diagnosis is the most probable?

Тромбоцитопатія Thrombocytopathy

Дефіцитна анемія Deficiency anemia

Лімфобластний лейкоз Lymphoblastic leukemia

Апластична анемія Aplastic anemia

Тромбоцитопенічна пурпура Thrombocytopenic purpura

26 / 130
Хворий 25-ти років надійшов через 2 години після дорожньо-транспортної аварії зі скаргами на постійний інтенсивний біль у правій паховій ділянці та у симфізі тазу, неможливість самостійно пересуватися. При огляді: положення хворого вимушене - поза ”жаби” (позитивний симптом Волковича). Садна й синці на шкірі в проекції симфізу та го-ризонтальної гілки лобкової кістки праворуч, виражений набряк м’яких тканин. Позитивний симптом ”прилиплої п’яти” праворуч. Поставте попередній діагноз: A 25-year-old patient arrived 2 hours after a traffic accident with complaints of constant intense pain in the right inguinal area and pelvic symphysis, inability to move independently. During the examination: the patient's position is forced - the 'frog' position (positive Volkovych's symptom). Bruises and bruises on the skin in the projection of the symphysis and the horizontal branch of the pubic bone on the right, pronounced swelling of soft tissues. Positive symptom of 'sticky heel' on the right Make a preliminary diagnosis:

Вивих правого стегна Dislocation of right hip

Травматична аневризма стегнової артерії Traumatic femoral artery aneurysm

Забій кісток тазу Pelvic fracture

Травматична пахвинна грижа Traumatic inguinal hernia

Закритий перелом лобкової кістки праворуч Closed fracture of pubic bone on the right

27 / 130
У породіллі через 3 тижні після пологів підвищилася температура тіла до 38°С, з’явилися остуда, слабкість та біль в лівій молочній залозі. Молочна залоза збільшена, нагрубла, болюча при пальпації; розм’якшення та флуктуації в ділянці інфільтрату немає. В крові - помірний лейкоцитоз. Який найбільш імовірний діагноз? 3 weeks after giving birth, the mother's body temperature rose to 38°C, chills, weakness and pain in the left mammary gland appeared. The mammary gland is enlarged, rough , painful on palpation; there is no softening and fluctuation in the area of the infiltrate. In the blood - moderate leukocytosis. What is the most likely diagnosis?

Серозний мастит Serous mastitis

Абсцедивний мастит Abscessive mastitis

Лактостаз Lactostasis

Іангренозний мастит Iangrenous mastitis

Мастопатія Mastopathy

28 / 130
В місті N проводилося вивчення захворюваності на інфаркт міокарда за попередні роки (з 2009 по 2013 роки). Який вид епідеміологічного дослідження був використаний? In the city N, the incidence of myocardial infarction was studied in previous years (from 2009 to 2013). What type of epidemiological study was used?

Експериментальний Experimental

Проспективний Prospective

Ретроспективний Retrospective

Аналітичний Analytic

Описовий Descriptive

29 / 130
Пацієнтка 20-ти років проходить лікування з приводу анемії (гемоглобін - 72 г/л). Півтора роки тому після мимовільного викидня у терміні 16 тижнів та крововтрати відзначає зниження пам’яті, втомлюваність, втрату апетиту, сухість шкіри, ламкість нігтів, набряклість, порушення менструальної функції. Об’єктивно: артеріальний тиск - 80/55 мм рт.ст., пульс - 54/хв., зріст - 168 см, вага - 48 кг, гіпоплазія статевих органів. Призначення якого з перерахованих препаратів буде найбільш доречним цій пацієнтці? A 20-year-old female patient is being treated for anemia (hemoglobin - 72 g/l). One and a half years ago, after an involuntary miscarriage at 16 weeks and blood loss, she noted a decrease memory loss, fatigue, loss of appetite, dry skin, brittle nails, swelling, menstrual dysfunction Objectively: blood pressure - 80/55 mm Hg, pulse - 54/min, height - 168 cm, weight - 48 kg, genital hypoplasia. Which of the listed drugs would be most appropriate for this patient?

Метотрексат Methotrexate

Гідроксихлорохін Hydroxychloroquine

Імуноглобулін людини нормальний Human immunoglobulin is normal

Інфліксимаб Infliximab

Гідрокортизон Hydrocortisone

30 / 130
В ході профогляду встановлено: у жінки 23-х років вагітностей не було. При бімануальному обстеженні виявлено: тіло матки нормальних розмірів, на передній стінці - щільне округле утворення на ніжці, пов’язане з маткою, розмірами в діаметрі до 6-ти см, неболюче, додатки без особливостей. За допомогою УЗД підтверджено діагноз суб- серозної міоми матки. Який метод лікування слід запропонувати? During the professional examination, it was established that a 23-year-old woman had no pregnancies. Bimanual examination revealed: the body of the uterus is of normal size, on the front wall - a dense rounded formation on legs, connected to the uterus, up to 6 cm in diameter, painless, appendages without features. The diagnosis of subserous uterine myoma was confirmed by ultrasound. What method of treatment should be offered?

Консервативна міомектомія Conservative myomectomy

Висока надпіхвова ампутація матки High supravaginal amputation of the uterus

Ампутація матки Uterine amputation

Екстирпація матки Uterus extirpation

Де фундація матки Where is the foundation of the uterus

31 / 130
Жінка 45-ти років скаржиться на колькоподібний біль у правому підребер’ї, що виникає після вживання жирної їжі, фізичного навантаження, іррадіює в праву лопатку та праве плече, зменшується при прийомі спазмолітиків. Хворіє впродовж року, періодично зазначає пожовтіння склер під час нападів болю. Який метод обстеження слід призначити в першу чергу для встановлення діагнозу? A 45-year-old woman complains of colic pain in the right hypochondrium, which occurs after eating fatty food, physical exertion, radiates to the right shoulder blade and right shoulder, decreases when taking antispasmodics. He has been sick for a year, periodically notes yellowing of the sclera during pain attacks. What examination method should be prescribed first of all to establish a diagnosis?

Фіброгастродуоденоскопію Fibrogastroduodenoscopy

Дуоденальне зондування Duodenal sounding

Комп’ютерну томографію Computed tomography

Ультрасонографію Ultrasonography

Рентгенографію органів черевної порожнини X-ray of abdominal organs

32 / 130
В ході обстеження дитини 5-ти років зі скаргами на постійний кашель з виділенням гнійного мокротиння та вологі хрипи справа у задньонижніх відділах виявлено: гнійний ендобронхіт ІІ-ІП ступеню, переважно справа, хлориди поту - 36 ммоль/л, на КТ - циліндричні бронхо- ектази в S9 та S10. Запропонуйте оптимальний метод лікування даної дитини: During the examination of a 5-year-old child with complaints of a constant cough with purulent sputum discharge and wet wheezing on the right in the lower back, it was found: purulent endobronchitis II-IP degree, mainly on the right, sweat chlorides - 36 mmol/l, on CT - cylindrical bronchiectasis in S9 and S10. Suggest the optimal method of treatment for this child:

Видалення уражених сегментів легень Removal of affected lung segments

Лаваж бронхіального дерева Bronchial tree lavage

Кінезотерапія Kinesiotherapy

Іенно-інженерна терапія Ion-engineering therapy

Тривала антибіотикотерапія Long-term antibiotic therapy

33 / 130
Хвора 25-ти років скаржиться на наявність декількох щільних, болючих вузлів у правій пахвовій ділянці, підвищення температури тіла до 38°С*, загальну слабкість. Хворіє 3 доби. Локально: в правій пахвовій області є три вузлоподібних утворення, які підвищуються над поверхнею шкіри, розмірами від 0,5 см до 1,5 см в діаметрі, різко болючі при пальпації, з чіткими контурами, шкіра над ними багрово-синюшного відтінку, підняття верхньої кінцівки обмежене через біль. Поставте діагноз: A 25-year-old patient complains of the presence of several dense, painful nodes in the right armpit, an increase in body temperature to 38°C*, general weakness. She has been ill for 3 days Locally: in the right axillary area there are three nodular formations that rise above the surface of the skin, the size of which is from 0.5 cm to 1.5 cm in diameter, sharply painful on palpation, with clear contours, the skin above them has a purplish-bluish tint, raising of the upper extremity is limited due to pain. Make a diagnosis:

Фурункул Furnish

Гнійний лімфаденіт Suppurative lymphadenitis

Абсцес Abscess

Гнійний гідраденіт Suppurative hidradenitis

Карбункул Carbuncle

34 / 130
Хворий скаржиться на почервоніння, печіння, відчуття стороннього тіла у правому оці. Захворів гостро. Під час огляду: гострота зору правого та лівого ока -1,0. У правому оці - гіперемія та набряк кон’юнктиви, поверхнева ін’єкція. У кон’юнктивальному мішку гнійні виділення. Рогівка прозора. Колір та малюнок райдужки не змінені, зіниця рухлива. Який найбільш імовірний діагноз? The patient complains of redness, burning, feeling of a foreign body in the right eye. He became acutely ill. During the examination: the visual acuity of the right and left eye is -1.0. in the right eye - hyperemia and swelling of the conjunctiva, superficial injection. Purulent discharge in the conjunctival sac. The cornea is transparent. The color and pattern of the iris have not changed, the pupil is mobile. What is the most likely diagnosis?

Гострий напад глаукоми Acute attack of glaucoma

Гострий кон’юнктивіт Acute conjunctivitis

Стороннє тіло рогівки Corneal foreign body

Гострий дакріоцистит Acute dacryocystitis

Гострий іридоцикліт Acute iridocyclitis

35 / 130
В організм людини з атмосферного повітря надходять декілька хімічних речовин. Як називається тип комбінованої дії, при якому сумісний ефект менший від суми ефектів кожної з речовин, що входить у комбінацію, при їх ізольованій дії на організм? Several chemical substances enter the human body from atmospheric air. What is the type of combined action called, in which the combined effect is less than the sum of the effects of each of the substances included in the combination , with their isolated action on the body?

Поєднана дія Combined Action

Антагонізм Antagonism

Комплексна дія Complex Action

ізольована дія isolated action

Потенціювання Potentiation

36 / 130
До лікаря звернувся чоловік 36-ти років зі скаргами на пекучий загрудин- ний біль, що зазвичай виникає через 13,5 години після вживання їжі. Зазначає, що біль підсилюється в горизонтальному положенні. При ендоскопії верхнього відділу ШКТ виявлені 2 вогнищевих ураження нижньої третини слизової оболонки стравоходу до 5 мм в межах однієї складки. Яка тактика лікаря буде найбільш доречною? A 36-year-old man went to the doctor with complaints of burning chest pain, which usually occurs 13.5 hours after eating. He notes that the pain intensifies in a horizontal position. Endoscopy of the upper part of the gastrointestinal tract revealed 2 focal lesions of the lower third of the esophageal mucosa up to 5 mm within one fold. What tactics of the doctor would be most appropriate?

- -

Призначення інгібіторів протонної помпи Prescription of proton pump inhibitors

Призначення ненаркотичних анальге- тиків Prescription of non-narcotic analgesics

Призначення кларитроміцину Clarithromycin Prescription

Хірургічне лікування Surgical treatment

37 / 130
Хворий 57-ми років перебуває на лікуванні з приводу політравми. Через 8 діб виникли скарги на виражений кашель з виділенням гнійного харкотиння, виражену задишку, підвищення температури до 38,8°С*. В анамнезі: хронічний бронхіт, палить 20 років. Об’єктивно: в легенях вислуховується жорстке дихання, в нижніх відділах правої легені - вологі дрібнопухирчасті хрипи. Проба на ВГЛ - негативна. Який діагноз у даного хворого? A 57-year-old patient is being treated for polytrauma. After 8 days, there were complaints of severe cough with purulent sputum, severe shortness of breath, temperature rise to 38, 8°C*. History: chronic bronchitis, smoker for 20 years. Objectively: hard breathing is heard in the lungs, moist fine-vesicular rales in the lower parts of the right lung. Test for VHL is negative. What is the diagnosis of this patient?

Нозокоміальна пневмонія Nosocomial pneumonia

інфекційне загострення бронхіту infectious exacerbation of bronchitis

Пневмонія осіб з імунодефіцитом Pneumonia of persons with immunodeficiency

Позагоспітальна пневмонія Community-acquired pneumonia

Аспіраційна пневмонія Aspiration pneumonia

38 / 130
Жінка 56-ти років звернулася до лікаря зі скаргами на підвищення температури до 38,5°С*, підвищену пітливість вночі та збільшення лімфовузлів в лівій пахвовій та лівій надключичній ділянках. У сімейному анамнезі рак молочної залози у бабусі. При фізикальному обстеженні в зазначених ділянках пальпуються збільшені, безболісні, щільні лімфовузли. При рентгенологічному дослідженні органів грудної порожнини розширення тіні середостіння за рахунок збільшених лімфовузлів. Після ексци- зійної біопсії лімфовузла надключичної ділянки, виявлені атипові багатоядерні клітини Рід-Штернберга. Який діагноз є найбільш імовірним? A 56-year-old woman consulted a doctor with complaints of a temperature rise to 38.5°С*, increased sweating at night and an increase in lymph nodes in the left axillary and left supraclavicular areas. There is a family history of breast cancer in the grandmother. During the physical examination, enlarged, painless, dense lymph nodes are palpated in the indicated areas. In the X-ray examination of the chest cavity, the mediastinal shadow is enlarged due to enlarged lymph nodes. After an excisional biopsy of the lymph node of the supraclavicular area, atypical multinucleated Reed-Sternberg cells. What is the most likely diagnosis?

Метастази раку молочної залози Breast cancer metastases

Лімфома Ходжкіна Hodgkin lymphoma

Туберкульоз Tuberculosis

Інфекційний мононуклеоз Infectious mononucleosis

Бронхогенна карцинома Bronchogenic carcinoma

39 / 130
На прийом до педіатра звернулась мати із 4-місячною дитиною зі скаргами на підвищення температури до 37,5°С. Дитина народжена доношеною, 8-9 балів за шкалою Апгар. При аналізі медичної документації виявлено, що у дитини немає щеплення проти туберкульозу. В які часові терміни повинно було бути проведене щеплення проти туберкульозу? A mother with a 4-month-old child came to the pediatrician with complaints of an increase in temperature to 37.5°C. The child was born full-term, 8-9 points on the scale Apgar. During the analysis of the medical records, it was found that the child has not been vaccinated against tuberculosis. When should the vaccination against tuberculosis have been carried out?

Доношена дитина, 3-5 доба життя Full-term baby, 3-5 days old

Протягом першого тижня життя During the first week of life

Протягом перебування в пологовому будинку після попередньої проби Манту During the stay in the maternity hospital after the previous Mantoux test

Протягом перших 24 годин життя During the first 24 hours of life

Доношена дитина, не пізніше 48 годин від народження Full-term baby, no later than 48 hours after birth

40 / 130
Жінка звернулась зі скаргами на мажучі кров’янисті виділення, біль в попереку. Остання менструація 2,5 місяці тому. При об’єктивному обстеженні: шийка матки ціанотична, бочкоподібна, розширена, зовнішнє вічко розташоване ексцентрично, матка збільшена як до 5 тижнів вагітності (пісочний годинник), придатки з обох боків не пальпу-ються, виділення з цервікального каналу кров’янисті, незначні. Яку патологію можна припустити? A woman complained of spotting bleeding, lower back pain. The last menstruation was 2.5 months ago. On objective examination: the cervix is cyanotic, barrel-shaped, expanded, the external eye is located eccentrically, the uterus is enlarged as before 5 weeks of pregnancy (hourglass), the appendages on both sides are not palpable, the discharge from the cervical canal is bloody, insignificant. What pathology can be assumed?

Субмукозний міоматозний вузол, що народжується Submucosal myomatous nodule at birth

Міома шийки матки Cervical myoma

Шийкова вагітність Cervical pregnancy

Поліп цервікального каналу Cervical canal polyp

Рак шийки матки Cervical cancer

41 / 130
До гінеколога звернулася жінка 36-ти років зі скаргами на нерегулярні мізерні менструації впродовж останніх 6 місяців, посилений ріст волосся на обличчі та ”огрубіння” голосу. В анамнезі два викидні, одні передчасні пологи. За да- ними УЗД матка та яєчники без особливостей. Яке обстеження доцільно провести для встановлення причини гормональних порушень? A 36-year-old woman came to the gynecologist with complaints of irregular scanty menstruation for the past 6 months, increased growth of facial hair and 'roughening' of the voice. She has two miscarriages, one premature birth. According to the ultrasound of the uterus and ovaries, there are no abnormalities. What examination should be performed to determine the cause of hormonal disorders?

Проведення рентген-дослідження турецького сідла Carrying out an X-ray examination of the Turkish saddle

Каріотипування Karyotyping

Проведення гістероскопи’ Hysteroscopy

Гормональне обстеження Hormonal examination

Взяття біопсії ендометрію Endometrial biopsy

42 / 130
У хірургічний кабінет звернувся хлопець 17-ти років зі скаргами на наявність рубаної рани правої стопи. Із анамнезу відомо, що травму отримав сокирою під час роботи по господарству. Всі щеплення отримував відповідно до віку. Об’єктивно: температура тіла - 36,8°С, пульс - 68/хв., артеріальний тиск - 120/75 мм рт.ст. Які дії лікаря? A 17-year-old boy came to the surgery room with complaints of a cut wound on his right foot. It is known from the anamnesis that he was injured with an ax while working on the farm. All received vaccinations according to age. Objectively: body temperature - 36.8°C, pulse - 68/min., blood pressure - 120/75 mm Hg. What actions did the doctor take?

Ввести правцевий анатоксин та протиправцевий імуноглобулін Enter tetanus toxoid and anti-tetanus immunoglobulin

Динамічне спостереження Dynamic monitoring

Ввести правцевий анатоксин Enter tetanus toxoid

Ввести правцевий анатоксин з дифтерійним анатоксином Enter tetanus toxoid with diphtheria toxoid

Ввести протиправцеву сироватку Enter anti-tetanus serum

43 / 130
В слабкорозвинених країнах Африки та Латинської Америки у дітей грудного віку можливий розвиток захворювання, для якого характерне відставання у фізичному та психічному розвитку, атрофія м’язів, жирова дистрофія печінки, депігментація волосся, гормональні порушення. Зміни у стані здоров’я розвиваються у випадку переводу дитини на штучне вигодовування переважно вуглеводною їжею з нестачею тваринних білків. Дане захворювання називається: In underdeveloped countries of Africa and Latin America, infants may develop a disease characterized by physical and mental retardation, muscle atrophy, fatty liver disease , hair depigmentation, hormonal disorders. Changes in the state of health develop in the case of transferring a child to artificial feeding with mainly carbohydrate food with a lack of animal proteins. This disease is called:

Хвороба Кешана Keshan's disease

Квашіоркор Kwashiorkor

Рахіт Rickets

Хвороба Прасада Prasad Disease

Аліментарна дистрофія Alimentary dystrophy

44 / 130
У лікаря-стоматолога 26-ти років при обстеженні виявили помірну гіперфер- ментемію, позитивний тест на HBsAg. Його дружина 24-х років не має щеплення від вірусного гепатиту В. Дитина 2-х років належно вакцинована за віковим графіком. Яка профілактика вірусного гепатиту В повинна бути проведена у родині? A 26-year-old dentist was diagnosed with moderate hyperenzymemia, a positive test for HBsAg. His 24-year-old wife is not vaccinated against viral hepatitis В. A 2-year-old child is properly vaccinated according to the age schedule. What prevention of viral hepatitis B should be carried out in the family?

Негайна вакцинація дружини при негативному тесті на HBsAg Immediate vaccination of wife with a negative HBsAg test

Призначення специфічного імуноглобуліну дружині та дитині Prescribing specific immunoglobulin to wife and child

Призначення специфічного імуноглобуліну дитині Prescribing specific immunoglobulin to a child

Призначення дружині специфічної хіміопрофілактики ненуклеозидними аналогами Prescribing specific chemoprophylaxis to the wife with non-nucleoside analogues

Призначення дружині та дитині специфічної хіміопрофілактики ненуклеозидними аналогами Appointment of specific chemoprophylaxis with non-nucleoside analogues to the wife and child

45 / 130
Після прогулянки на вулиці в ясний сонячний день у дитини 8 місяців з проявами рахіту II ступеню, підгострого перебігу, періоду розпалу, з’явилися судоми у вигляді карпопедального спазму. Вони свідчать про наявність у дитини: After a walk on the street on a clear sunny day, an 8-month-old child with manifestations of rickets of the II degree, subacute course, in the period of exacerbation, developed convulsions in the form of carpopedal spasm. They indicate that the child has:

Спазмофілії Spasmophilia

Епілепсії Epilepsy

Менінгіту Meningitis

Черепно-мозкової травми Traumatic brain injury

ГРВІ SARS

46 / 130
Хвора 42-х років скаржиться на наявність болючого утворення у лівій сідниці, підвищення температури тіла до 38,2°С. Тиждень тому завершила курс лікування з приводу поперекового радикуліту. У верхньо-зовнішньому квадранті лівої сідниці гіперемія, набряк. При пальпації - різко болюче ущільнення до 6 см в діаметрі з розм’якшенням у центрі. Ваш попередній діагноз: A 42-year-old patient complains of the presence of a painful mass in the left buttock, an increase in body temperature to 38.2°С*. A week ago, she completed a course of treatment with due to lumbar radiculitis. Hyperemia, swelling in the upper-outer quadrant of the left buttock. Upon palpation, there is a sharply painful compaction up to 6 cm in diameter with softening in the center. Your previous diagnosis:

Нагноєна атерома Suppurative atheroma

Підшкірний парапроктит Subcutaneous paraproctitis

Післяін’єкційний абсцес After Action Abscess

Абсцедивний фурункул Abscess boil

Карбункул сідниці Carbuncle of the buttock

47 / 130
До приймального відділення звернулася хвора 28-ми років зі скаргами на біль в ділянці лівого плечового суглобу, який посилюється при рухах. При огляді: в ділянці ключично-акроміального з’єднання виявляється набряк і сходин- коподібна деформація. При пальпації - болючість, позитивний симптом ”клавіші”. Який попередній діагноз? A 28-year-old female patient came to the emergency department with complaints of pain in the area of the left shoulder joint, which worsens with movement. On examination: in the area of the clavicular-acromial with swelling and a step-like deformation are found in the joint. On palpation - soreness, a positive 'key' symptom. What is the previous diagnosis?

Перелом головки плечової кістки Fracture of humeral head

Перелом акроміального відростка лопатки Fracture of the acromial process of the scapula

Передньо-верхній вивих плечової кістки Anterior-superior dislocation of the humerus

Вивих акроміального кінця ключиці Dislocation of the acromial end of the clavicle

Перелом акроміального кінця ключиці Fracture of the acromial end of the clavicle

48 / 130
На прийом до лікаря акушера- гінеколога жіночої консультації звернулася жінка зі строком вагітності 10 тижнів (перше звертання). Який з медичних документів повинен бути заведений лікарем у цьому випадку для контролю за перебігом вагітності? A woman with a 10-week pregnancy period applied to see an obstetrician-gynecologist at the women's consultation (first visit). Which of the medical documents should be prepared by the doctor in this case for pregnancy control?

Індивідуальна карта вагітної, породіллі Individual card of a pregnant woman, a woman giving birth

Контрольна карта диспансерного спостереження Dispensary monitoring control card

Медична карта амбулаторного хворого Medical card of an outpatient

Карта обліку диспансеризації Dispensary accounting card

Історія пологів History of childbirth

49 / 130
У пацієнта 42-х років виник ”кинджальний” біль в епігастральній ділянці, що поширився через кілька годин над всією поверхнею живота. Симптоми подразнення очеревини позитивні. Протягом багатьох років страждає на виразкову хворобу дванадцятипалої кишки. При рентгенографії живота: вільний газ під куполами діафрагми. Ваш імовірний діагноз: A 42-year-old patient developed a 'dagger' pain in the epigastric region, which spread after a few hours over the entire surface of the abdomen. The symptoms of peritoneal irritation are positive. For many years suffering from duodenal ulcer. Abdominal x-ray: free gas under the domes of the diaphragm. Your probable diagnosis:

Перфорація порожнистого органу. Перитоніт Perforation of a hollow organ. Peritonitis

Виразкова хвороба дванадцятипалої кишки Duodenal ulcer

Виразкова хвороба шлунка в стадії загострення Gastric ulcer disease in the acute stage

Хронічний гастрит Chronic gastritis

Іострий панкреатит Acute pancreatitis

50 / 130
В пологовий будинок доставлена вагітна 32-х років. Зі слів родичів дома мала судоми, втратила свідомість. Турбує головний біль, загальна слабкість, свідомість дещо затьмарена. Вагітність II, 34 тижні. AT- 170/120 мм рт.ст., пульс - 100/хв. Спостерігаються генералізовані набряки, що виникли 3 тижні тому. Серцебиття плода - ритмічне, приглушене, 124/хв. Яка тактика? A 32-year-old pregnant woman was taken to the maternity hospital. According to relatives, she had convulsions at home and lost consciousness. She is bothered by a headache, general weakness, consciousness is somewhat clouded. Pregnancy II , 34 weeks. AT- 170/120 mm Hg, pulse - 100/min. Generalized swellings that occurred 3 weeks ago are observed. Fetal heartbeat - rhythmic, muffled, 124/min. What are the tactics?

Інтенсивна терапія, родорозрішення впродовж доби Intensive therapy, delivery during the day

Інтенсивна терапія, родорозрішення в терміні 36 тижнів Intensive therapy, delivery within 36 weeks

Інтенсивна терапія, родорозрішення в терміні 37-40 тижнів Intensive therapy, delivery within 37-40 weeks

Інтенсивна терапія, родорозрішення впродовж тижня Intensive therapy, delivery within a week

Інтенсивна терапія та негайне родоро- зрішення Intensive therapy and immediate delivery

51 / 130
У дівчинки 11-ти років (в анамнезі часті ангіни) захворювання мало поступовий розвиток. З’явились дратівливість, неуважність, гіперкінези. При обстеженні виявлено порушення координації рухів, гіпотонію м’язів, гіперрефлексію. Для якого захворювання характерні такі зміни зі сторони центральної нервової системи? In an 11-year-old girl (with a history of frequent angina), the disease had a gradual development. Irritability, inattention, hyperkinesis appeared. During the examination, a violation of movement coordination was found, muscle hypotonia, hyperreflexia. What disease is characterized by such changes on the part of the central nervous system?

Іостра ревматична лихоманка Acute rheumatic fever

Системна склеродермія Systemic scleroderma

Системний червоний вовчак Systemic lupus erythematosus

Вегето-судинна дисфункція Vegeto-vascular dysfunction

Вузликовий поліартеріїт Polyarteritis nodosa

52 / 130
Породілля на 11-ту добу після пологів поскаржилась на різкий біль у лівій молочній залозі, підвищення температури тіла до 39,0°С*. Патології з боку внутрішніх та статевих органів не виявлено. Ліва молочна залоза гаряча на дотик, болюча. У верхньо-зовнішньому квадранті шкіра гіперемована, дещо ціа- нотична, набрякла. При пальпації визначається інфільтрат 6x8 см з флуктуацією посередині. Ваша тактика ведення: On the 11th day after giving birth, the woman in labor complained of sharp pain in the left mammary gland, an increase in body temperature to 39.0°C*. Pathologies from the internal and no genital organs were detected. The left mammary gland is hot to the touch, painful. In the upper-external quadrant, the skin is hyperemic, somewhat cyanotic, swollen. Palpation reveals an infiltrate of 6x8 cm with fluctuation in the middle. Your management tactics:

Хірургічне лікування маститу Surgical treatment of mastitis

Припинити лактацію агоністами до- фаміну Stop lactation with dopamine agonists

Рекомендувати продовжити грудне годування Recommend to continue breastfeeding

Рекомендувати зціджування молока Recommend expressing milk

Антибактеріальна терапія Antibacterial therapy

53 / 130
Роділля, І вагітність, 38 тижнів, скаржиться на нерегулярний переймоподібний біль внизу живота та в попереку протягом 2 діб, вночі спала погано. При зовнішньому акушерському дослідженні - тонус матки підвищений, поздовжнє положення плода І позиція, передній вид, голівка плода рухома над входом в малий таз. Піхвове дослідження - шийка матки до 2,5 см, відхилена дозаду, розм’якшена нерівномірно, зовнішнє вічко відкрите до 0,5 см, внутрішнє вічко закрите. Ваш діагноз: Mother in labor, 1st pregnancy, 38 weeks, complains of irregular spasm-like pain in the lower abdomen and lower back for 2 days, she slept poorly at night. During external obstetric examination - uterine tone elevated, longitudinal position of the fetus I position, front view, the head of the fetus is mobile above the entrance to the small pelvis. Vaginal examination - the cervix is up to 2.5 cm, deviated backward, softened unevenly, the external eye is open to 0.5 cm, the internal eye closed Your diagnosis:

Дискоординована пологова діяльність Discoordinated birth activity

Первинна слабкість пологової діяльності Primary weakness of labor activity

Патологічний прелімінарний період Pathological preliminary period

Вторинна слабкість пологової діяльності Secondary weakness of labor activity

Передвісники пологів Forerunners of childbirth

54 / 130
У хворої 30-ти років припинилась менструація, а потім почала знижуватися гострота зору. Була виявлена первинна атрофія зорових нервів, бітемпо- ральна геміанопсія. Впродовж 2-х років хвора лікувалась окулістами амбулаторно та стаціонарно, але зір прогресивно падав. Консультація офтальмолога: гострота зору OD=0,02, OS= 0,03, вира-жена первинна атрофія зорових нервів. Який допоміжний метод обстеження може допомогти у постановці діагнозу? A 30-year-old patient stopped menstruating, and then her visual acuity began to decrease. Primary atrophy of the optic nerves, bitemporal hemianopsia was detected. Over the course of 2 years the patient was treated by ophthalmologists on an outpatient and inpatient basis, but her vision progressively decreased. Ophthalmologist's consultation: visual acuity OD=0.02, OS= 0.03, pronounced primary atrophy of the optic nerves. What auxiliary method of examination can help in making a diagnosis?

Ехоенцефалографія Echoencephalography

Дослідження ліквору Liquor Research

Пневмоенцефалографія Pneumoencephalography

Електроенцефалографія Electroencephalography

Рентгенографія черепу X-ray skull

55 / 130
До оториноларинголога звернувся хворий 29-ти років зі скаргами на зниження слуху При аудіологічному обстеженні виявлено порушення звукопрове- дення, кісткова провідність не порушена. Про порушення роботи яких структур органу слуху йде мова? A 29-year-old patient turned to an otorhinolaryngologist with complaints of hearing loss. During the audiological examination, a sound conduction disorder was detected, bone conduction was not impaired. About the dysfunction of which organ structures is it about hearing?

Основи завитки та довгастого мозку Fundamentals of gyrus and medulla oblongata

Системи півколових каналів Semicircular canal systems

Спірального органу Spiral Organ

Барабанної перетинки, ланцюга слухових кісточок Tympanic membrane, chain of auditory ossicles

Отолітового апарату Otolith apparatus

56 / 130
У жінки 34-х років з попередньо нормальною менструальною функцією цикли стали нерегулярними, за даними тестів функціональної діагностики - анову- ляторними. Молочні залози пальпатор- но болячі, виділяється молоко (галакто- рея). Яке дослідження показано хворій в першу чергу? In a 34-year-old woman with previously normal menstrual function, the cycles became irregular, according to functional diagnostic tests - anovulatory. The mammary glands are painful on palpation, discharge milk (galactorrhoea). What research is shown to the patient in the first place?

Прогестеронова проба Progesterone test

КТ головного мозку CT brain

Ультразвукове дослідження органів малого тазу Ultrasound examination of pelvic organs

Визначення рівня пролактину Prolactin level determination

Визначення рівня гонадотропінів Determining the level of gonadotropins

57 / 130
Сімейний лікар вирішив зайнятися підприємницькою діяльність в сфері надання первинної медичної допомоги населенню. Яку організаційно-правову форму підприємництва доцільно обрати? The family doctor decided to start a business in the field of providing primary medical care to the population. What organizational and legal form of business should be chosen?

Підприємство зі створенням юридичної особи Enterprise with the creation of a legal entity

Акціонерне товариство Joint joint stock company

Товариство з повною відповідальністю General Liability Company

Приватний підприємець без створення юридичної особи Private entrepreneur without creating a legal entity

Товариство з обмеженою відповідальністю Limited Liability Company

58 / 130
Головний лікар ЦПМСД має надати звіт щодо забезпеченості населення лікарями. За допомогою якого коефіцієнту він може розрахувати даний показник? The chief physician of the CPMSD must submit a report on the provision of doctors to the population. By which coefficient can he calculate this indicator?

Співвідношення Ratio

Наочності Visibility

Інтенсивності Intensities

Екстенсивності Extensives

Спеціальної інтенсивності Special intensity

59 / 130
Анестезіолог проводить наркоз під час ургентної операції з приводу розлитого перитоніту жінці 65-ти років вагою 135 кг. Спроби інтубації трахеї після введення міорелаксантів тричі поспіль виявились невдалими. Штучна вентиляція легень через лицьову маску ефективна. Яким чином забезпечити прохідність дихальних шляхів і проведення штучної вентиляції легень? The anesthesiologist administers anesthesia during an emergency operation for diffuse peritonitis to a 65-year-old woman weighing 135 kg. Attempts to intubate the trachea after the administration of muscle relaxants were unsuccessful three times in a row. Artificial ventilation lungs through a face mask is effective. How to ensure patency of the respiratory tract and artificial ventilation of the lungs?

Проводити штучну вентиляцію легень, встановивши орофарингеальний повітровід Perform artificial lung ventilation by installing an oropharyngeal airway

Проводити штучну вентиляцію легень через назальні канюлі Perform artificial lung ventilation through nasal cannulas

Проводити штучну вентиляцію легень через лицьову маску Perform artificial lung ventilation through a face mask

Проводити штучну вентиляцію легень, встановивши назофарингеальний повітровід Carry out artificial lung ventilation by installing a nasopharyngeal airway

Проводити штучну вентиляцію легень, встановивши ларингеальну маску Carry out artificial lung ventilation by installing a laryngeal mask

60 / 130
У приймальне відділення звернулися батьки з двомісячним хлопчиком, який напередодні ввечері впав із дивана на підлогу. Об’єктивно: дитина не контактує, млява, мали місце тоніко-клонічні судоми, в скроневій ділянці зліва напружена гематома. Яке із досліджень слід провести в першу чергу? Parents of a two-month-old boy who fell from the sofa to the floor the night before came to the reception department. Objectively: the child is unresponsive, lethargic, tonic-clonic seizures occurred convulsions, tense hematoma in the temporal region on the left. Which of the studies should be performed first?

Іонограма сироватки крові Ionogram of blood serum

Комп’ютерна томограма голови Computer tomography of the head

Електроенцефалографія Electroencephalography

Дослідження ліквору Liquor Research

Оглядова рентгенографія голови Overview radiography of the head

61 / 130
Хлопчик 11-ти років хворіє на бронхіальну астму. На тлі ГРВІ з’явилася ядуха, сухий кашель, свистяче дихання. Пацієнт впродовж години тричі з інтервалом у 20 хвилин отримував інгаляції бета-2-агоніста короткої дії та ан- тихолінергічного засобу, проте стан не покращився. Наступним кроком невідкладної допомоги є призначення: An 11-year-old boy suffers from bronchial asthma. On the background of SARS, dyspnea, dry cough, wheezing appeared. The patient three times in an hour with an interval of 20 minutes received inhalations of a short-acting beta-2 agonist and an anticholinergic agent, but the condition did not improve. The next step in emergency care is to prescribe:

Муколітичних препаратів Mucolytic drugs

Магнезії сульфату Magnesium sulfate

Метилксантинів короткої дії Short-acting methylxanthines

Системних глюкокортикостероїдів Systemic glucocorticosteroids

Седативних препаратів Sedative drugs

62 / 130
У хворої 19-ти років слабкість, помірний головний біль, біль у горлі при ковтанні. Хворіє 3 дні. Об’єктивно: температура тіла - 38,2°С*, генералізована лімфоаденопатія. Ознаки тонзилофа- рингіту. Помірна гепатоспленомегалія. ЗАК: 35% атипових мононуклеарів. Яке дослідження необхідно призначити для верифікації діагнозу? A 19-year-old patient has weakness, moderate headache, sore throat when swallowing. She has been ill for 3 days. Objectively: body temperature - 38.2° C*, generalized lymphadenopathy. Signs of tonsillopharyngitis. Moderate hepatosplenomegaly. SAC: 35% of atypical mononuclear cells. What research should be prescribed to verify the diagnosis?

Дослідження крові на IgM anti-VCA EBV Blood test for IgM anti-VCA EBV

Дослідження крові на сумарні антитіла до ВІЛ Blood test for total HIV antibodies

Дослідження крові на IgM anti-CMV Blood test for IgM anti-CMV

Дослідження крові на IgM anti-HHV-6 Blood test for IgM anti-HHV-6

Дослідження крові на IgM anti-HHV-7 Blood test for IgM anti-HHV-7

63 / 130
Хвора 48-ми років доставлена в лікарню з неадекватною поведінкою. Зі слів чоловіка, постійно вживала знеболювальні препарати у зв’язку з мігренню та болем у спині. Об’єктивно: жов- тяничність склер, чутливість у правому верхньому квадранті живота та астери- ксис. БАК: АЛТ- 649 Од/л, загальний білірубін - 95,8 мкмоль/л та МНС- 6,8. Який найбільш імовірний діагноз? A 48-year-old patient was brought to the hospital with inappropriate behavior. According to her husband, she was constantly taking painkillers due to migraines and back pain. About' objectively: scleral jaundice, sensitivity in the right upper quadrant of the abdomen and asterixis. BAC: ALT - 649 U/l, total bilirubin - 95.8 μmol/l and MHC - 6.8. What is the most likely diagnosis?

Іостра наднирникова недостатність Acute adrenal insufficiency

Інтоксикація знеболювальними засобами Intoxication with painkillers

Іостра печінкова недостатність Acute liver failure

Іостра ниркова недостатність Acute renal failure

Іостре порушення мозкового кровообігу Acute cerebrovascular accident

64 / 130
У жінки 32-х років скарги на двосторонній біль у руках і припухлість пальців, відчуття скутості впродовж кількох годин після пробудження, втомлюваність. Об’єктивно: набряк другого та третього п’ястково-фалангових і проксимальних міжфалангових суглобів, позитивні симптоми поперечного стискання кисті. Яке захворювання найбільш імовірне? A 32-year-old woman complains of bilateral hand pain and swelling of the fingers, a feeling of stiffness for several hours after waking up, fatigue. Objectively: swelling of the second and third metacarpal-phalangeal and proximal interphalangeal joints, positive symptoms of transverse compression of the hand. What disease is most likely?

Подагричний артрит Gouty arthritis

Ревматоїдний артрит Rheumatoid arthritis

Ревматичний артрит Rheumatic arthritis

Реактивний артрит Reactive arthritis

Остеоатрит Osteoarthritis

65 / 130
Хворий 38-ми років скаржиться на періодичну висипку в ділянці бороди і вусів, що супроводжується помірною болючістю. Хворіє впродовж 2-х років. Об’єктивно: в ділянці бороди та вусів на фоні гіперемії та помірної інфільтрації - множинні згруповані пустульозні елементи, дрібні ерозії та гнійні кірки. На яке супутнє захворювання слід обстежити пацієнта в першу чергу? A 38-year-old patient complains of a periodic rash in the area of the beard and mustache, which is accompanied by moderate pain. He has been ill for 2 years. Objectively: in the area beards and mustaches on the background of hyperemia and moderate infiltration - multiple grouped pustular elements, small erosions and purulent crusts. For which concomitant disease should the patient be examined first of all?

Туберкульозна інфекція Tuberculosis infection

Іепатит С Hepatitis C

Цукровий діабет Diabetes

Захворювання щитоподібної залози Thyroid disease

Дисбіоз кишечника Intestinal dysbiosis

66 / 130
Хворому 65-ти років з приводу застійної серцевої недостатності призначена фармакотерапія (фуросемід, спіро- нолактон, бісопролол, раміприл). Через 2 місяці звернувся до лікаря з приводу розвитку гінекомастїї. Запідозрено побічну дію спіронолактону. На який з наведених препаратів можна його замінити у схемі лікування цього хворого? A 65-year-old patient was prescribed pharmacotherapy (furosemide, spironolactone, bisoprolol, ramipril) for congestive heart failure. After 2 months, he consulted a doctor about the development gynecomastia. A side effect of spironolactone is suspected. Which of the following drugs can replace it in the treatment regimen of this patient?

Еплеренон Eplerenone

Ніфедипін Nifedipine

Торасемід Torasemide

Івабрадин Ivabradin

Триметазидин Trimetazidine

67 / 130
У хворої 31-го року після проведеної субтотальної резекції щитоподібної залози з приводу токсичного зоба виникли серцебиття, пітливість, психомоторне збудження. Пацієнтка знепритомніла. Об’єктивно: температура тіла - 39,7°С*, частота дихання - 38/хв.; ЧСС- 135/хв., миготлива аритмія. AT- 190/80 мм рт.ст. Імовірне ускладнення: A 31-year-old patient developed palpitations, sweating, psychomotor agitation after subtotal resection of the thyroid gland for toxic goiter. The patient fainted. Objectively: body temperature - 39.7°С*, respiratory rate - 38/min.; Heart rate - 135/min., atrial fibrillation. AT - 190/80 mm Hg. Probable complication:

Істеричний напад Hysterical attack

Гіпертонічний криз Hypertensive crisis

Тиреотоксичний криз Thyrotoxic crisis

Бактеріальний шок Bacterial shock

Рецидив дифузного токсичного зоба Recurrence of diffuse toxic goiter

68 / 130
Чоловік 48-ми років скаржиться на стискаючий біль за грудиною, що виник вперше 1,5 години тому після фізичного навантаження та не знімається нітрогліцерином. Об’єктивно: ЧСС- 75/хв., ЧД- 16/хв., AT- 140/80 мм рт.ст. При ЕКГ дослідженні сегмент ST зміщений донизу від ізолінії на 1-2 мм у відведеннях V4- V6. Дослідження якого показника допоможе встановити діагноз у даному випадку? A 48-year-old man complains of squeezing pain behind the sternum, which first appeared 1.5 hours ago after physical exertion and is not relieved by nitroglycerin. Objectively: Heart rate - 75/min., HR- 16/min., AT- 140/80 mm Hg. During an ECG examination, the ST segment is shifted downward from the isoline by 1-2 mm in leads V4-V6. The study of which indicator will help to establish a diagnosis in in this case?

Натрій уретичного пептиду Uretic peptide sodium

Тропоніну Troponin

КФК KFC

АсАТ АСАТ

D-димеру D-dimer

69 / 130
Жінка 50-ти років, яка страждає на системний червоний вовчак, отримує в якості базисної терапії метотрексат 10 мг на тиждень та метипред у дозі 8 мг на добу впродовж останнього року. Не вакцинована з підліткового віку, бажає отримати щеплення від дифтерії. Яка тактика імунопрофілактики у даному випадку? A 50-year-old woman with systemic lupus erythematosus receives as basic therapy methotrexate 10 mg per week and metipred 8 mg per day during the last year. Has not been vaccinated since adolescence, wants to be vaccinated against diphtheria. What are the tactics of immunoprophylaxis in this case?

Відмінити імуносупресивну терапію та провести щеплення Cancel immunosuppressive therapy and carry out vaccination

При відсутності антитіл щеплення після відміни імуносупресивної терапії In the absence of vaccination antibodies after withdrawal of immunosuppressive therapy

Пасивна імунопрофілактика протидифтерійним імуноглобуліном Passive immunoprophylaxis with diphtheria immunoglobulin

Щеплення за стандартною схемою Vaccination according to the standard scheme

При відсутності антитіл щеплення за стандартною схемою In the absence of antibodies, vaccination according to the standard scheme

70 / 130
Хворого 46-ти років турбують часті напади ядухи, порушення сну через пароксизми експіраторної задишки 35 разів на тиждень, обмеження фізичної активності. Об’єктивно: везикулярне дихання з подовженим видихом, сухі свистячі хрипи на видиху Спірометрія: ОФВ1- 67%, позитивна проба з бета- 2-агоністом, добові коливання ПОШВИД більше 30%. Якою має бути контролююча терапія даного хворого? A 46-year-old patient is bothered by frequent attacks of dyspnea, sleep disturbances due to paroxysms of expiratory shortness of breath 35 times a week, limitations in physical activity. Objectively: vesicular breathing with prolonged exhalation, dry whistling wheezes on exhalation Spirometry: FEV1- 67%, positive test with beta-2-agonist, daily fluctuations of PHOSHVID more than 30%. What should be the control therapy of this patient?

Тіотропій + формотерол Tiotropium + formoterol

Фенспірид + сальбутамол Fenspiride + Salbutamol

Іпратропій + фенотерол Ipratropium + Fenoterol

Флютиказон + сальметерол Fluticasone + Salmeterol

Іпратропій + сальбутамол Ipratropium + salbutamol

71 / 130
У хворої 34-х років сильний біль глибоко в орбіті, головний біль, підвищення температури тіла. Захворювання виникло гостро. Об’єктивно: повіки набряклі, шкіра червоного кольору, екзофтальм, обмеження рухів очного яблука. Ваш діагноз: A 34-year-old patient has severe pain deep in the orbit, headache, increased body temperature. The disease occurred acutely. Objectively: the eyelids are swollen, the skin is red , exophthalmos, restriction of movements of the eyeball. Your diagnosis:

Абсцес повіки Eyelid abscess

Ретробульбарний неврит Retrobulbar neuritis

Блефарит Blepharitis

Флегмона орбіти Phlegmon of the orbit

Виразка рогівки Corneal ulcer

72 / 130
Хвора скаржиться на підвищення температури тіла до 38°С* протягом 2х діб. Об’єктивно: позитивний симптом постукування в поперековій ділянці зліва. Загальний аналіз сечі: питома вага - 1015, білок - 0,99 г/л, еритроцити - 6-8 в п/з, лейкоцити - 30-32 в п/з. Який попередній діагноз? The patient complains of an increase in body temperature up to 38°С* for 2 days. Objectively: a positive symptom of tapping in the lumbar region on the left. General analysis of urine: specific gravity - 1015, protein - 0.99 g/l, erythrocytes - 6-8 in p/z, leukocytes - 30-32 in p/z. What is the previous diagnosis?

Гострий цистит Acute cystitis

Гострий пієлонефрит Acute pyelonephritis

Сечокам’яна хвороба Urolithiasis

Гострий гломерулонефрит Acute glomerulonephritis

Амілоїдоз нирок Kidney amyloidosis

73 / 130
Хворий скаржиться на підвищення температури тіла до 39,4°С*, головний біль, блювання. Ригідність м’язів потилиці, симптом Керніга позитивний, вогнищевої симптоматики немає. Ліквор: цитоз -19600 мкл, лімфоцити - 27%, нейтрофіли - 73%, білок - 6,3 г/л. Імовірний діагноз: The patient complains of an increase in body temperature up to 39.4°C*, headache, vomiting. Neck muscle stiffness, Kernig's symptom is positive, there are no focal symptoms. CSF: cytosis - 19,600 μl, lymphocytes - 27%, neutrophils - 73%, protein - 6.3 g/l. Probable diagnosis:

Менінгококовий менінгіт Meningococcal meningitis

Туберкульозний менінгіт Tuberculous meningitis

Ентеровірусний менінгіт Enterovirus meningitis

Субарахноїдальний крововилив Subarachnoid hemorrhage

Герпетичний менінгіт Herpetic meningitis

74 / 130
Хворий 46-ти років госпіталізований в інфекційне відділення з діагнозом: гострий вірусний гепатит B. На цукровий діабет хворіє 9 років, дотримується дієти, отримує метформін - 2 г/добу, глімепірид - 2 мг вранці. Цукор крові - 7,3 ммоль/л. Іліколізований гемоглобін -6,8%. Яка подальша тактика лікування хворого? A 46-year-old patient was hospitalized in the infectious department with a diagnosis of acute viral hepatitis ','

Залишити терапію без змін Leave therapy unchanged

Перевести хворого на інсулін Transfer patient to insulin

Збільшити дозу метформіну Increase metformin dose

Збільшити дозу глімепіриду He has been suffering from diabetes for 9 years, follows a diet, receives metformin - 2 g/day, glimepiride - 2 mg in the morning. Blood sugar - 7.3 mmol /l. Ilycolyzed hemoglobin -6.8%. What is the further tactics of treating the patient?

Перевести хворого на глібенкламід Transfer the patient to glibenclamide 6- Increase the dose of glimepiride

75 / 130
Хвора 42-х років після відпочинку, встала з ліжка і відчула загальну слабкість, запаморочення, потемніння в очах, втратила свідомість. Об’єктивно: хвора непритомна, шкіра бліда, холодна, зіничні та сухожильні рефлекси збережені. AT- 75/50 мм рт.ст., пульс - 100/хв. Який діагноз найбільш імовірний? A 42-year-old patient after rest, got out of bed and felt general weakness, dizziness, darkening of the eyes, lost consciousness. Objectively: the patient is unconscious, the skin pale, cold, pupillary and tendon reflexes are preserved. AT - 75/50 mm Hg, pulse - 100/min. What is the most likely diagnosis?

Ішемічний інсульт Ischemic stroke

Епілептичний синдром Epileptic syndrome

Істеричний напад Hysterical attack

Гіпоглікемічна кома Hypoglycemic coma

Ортостатичний колапс Orthostatic collapse

76 / 130
Хворий 35-ти років звернувся до сімейного лікаря з раптовим кинджальним болем в епігастрії; в анамнезі виразка шлунку. Об’єктивно: дошкоподібне напруження м’язів передньої черевної стінки, позитивний симптом Щоткіна- Блюмберга. При рентгенологічному дослідженні виявлено серпоподібне просвітлення під склепінням діафрагми. Яке ускладнення розвинулося? A 35-year-old patient went to the family doctor with a sudden stabbing pain in the epigastrium; a history of stomach ulcer. Objectively: plate-like tension of the muscles of the anterior abdominal wall , a positive symptom of Shtokkin-Blumberg. During the X-ray examination, a sickle-shaped light was found under the vault of the diaphragm. What complication developed?

Малігнізація виразки шлунку Malignancy of gastric ulcer

Пенетрація виразки шлунку Stomach ulcer penetration

Стеноз воротаря Stenosis of the goalkeeper

Перфорація виразки шлунку Perforation of gastric ulcer

Виразкова кровотеча Ulcer bleeding

77 / 130
У хлопчика 3-х років під час ходьби та бігу збільшується права половина мошонки, яка в положенні лежачи зменшується до нормальних розмірів. Діагноз: In a 3-year-old boy, the right half of the scrotum increases during walking and running, which decreases to normal size when lying down. Diagnosis:

Сполучна водянка оболонок правого яєчка Conjunctive dropsy of the membranes of the right testicle

Орхіепідидиміт Orchiepididymitis

Цистоцеле Cystocele

Парафімоз Paraphimosis

Фімоз Phimosis

78 / 130
Під час судово-медичної експертизи трупа судово-медичний експерт описав у тім’яно-скроневій ділянці праворуч рану лінійної форми, розміром 6,4 см при зведених краях; краї нерівні, осаднені, в глибині рани видно тканинні перетинки. Дайте назву описаної рани: During the forensic medical examination of the corpse, the forensic medical expert described a wound of a linear shape in the right parietal-temporal area, the size of 6.4 cm with folded edges; the edges are uneven, precipitated, in the depth of the wound tissue membranes are visible. Give the name of the described wound:

Забита рана Stacked wound

Рубана рана Cutting wound

Колота рана Puncture wound

Розсічена рана Cut wound

Різана рана Cut wound

79 / 130
Лікар швидкої допомоги під час надання медичної допомоги особі, витягнутій із зашморга родичами, відзначив: відсутність пульсу на сонних артеріях, свідомості, самостійного дихання, корнеа- льних рефлексів та наявність трупних плям на спині і задній поверхні кінцівок. За якими ознаками можна констатувати настання смерті? The emergency physician, while providing medical assistance to a person who was pulled out of a coma by relatives, noted: the absence of a pulse on the carotid arteries, consciousness, independent breathing, corneal reflexes and the presence of cadaveric spots on the back and back surface of the limbs. What signs can be used to determine the onset of death?

Наявність трупних плям Presence of corpse stains

Відсутність самостійного дихання Lack of spontaneous breathing

Відсутність пульсу No pulse

Відсутність свідомості Lack of consciousness

Відсутність корнеальних рефлексів Absence of corneal reflexes

80 / 130
Дівчинка 4-х років захворіла гостро після перенесеної 3 тижні тому стрептококової інфекції на шкірі. З’явилися набряки обличчя, нижніх кінцівок, головний біль, темний колір сечі (колір ”кока- коли”). AT- 125/60 мм рт.ст. Аналіз сечі: білок - 3,3°/ОО, питома вага -1012, лейкоцити - 1-3 в п/з, еритроцити вкривають все п/з. Добова протеїнурія - 980 мг. Діурез - 550 мл. Загальний білок крові - 60 г/л. Альбуміни - 55%. Холестерин - 4,5 ммоль/л. Креатинін сироватки - 89 мкмоль/л. Який найбільш імовірний діагноз? A 4-year-old girl became acutely ill after suffering a streptococcal infection on the skin 3 weeks ago. Swelling of the face, lower limbs, headache, dark urine color appeared ( color 'Coca-Cola'). AT- 125/60 mm Hg Urinalysis: protein - 3.3°/OO, specific gravity -1012, leukocytes - 1-3 in p/z, erythrocytes cover all p/ z. Daily proteinuria - 980 mg. Diuresis - 550 ml. Total blood protein - 60 g/l. Albumin - 55%. Cholesterol - 4.5 mmol/l. Serum creatinine - 89 μmol/l. What is the most likely diagnosis?

Гломерулонефрит, нефритичний синдром Glomerulonephritis, nephritic syndrome

Гломерулонефрит, нефротичний синдром Glomerulonephritis, nephrotic syndrome

Сечокам’яна хвороба Urolithiasis

Пієлонефрит Pyelonephritis

Гостре ураження нирок Acute kidney injury

81 / 130
У вогнищах облисіння на голові волосся обламане на рівні 4-6 мм від поверхні шкіри, відзначається гіперемія, мукоподібне лущення. Імовірний попередній діагноз? In foci of baldness on the head, the hair is broken off at the level of 4-6 mm from the surface of the skin, there is hyperemia, muco-like desquamation. Possible preliminary diagnosis?

Себорейне облисіння Seborrheic alopecia

Мікроспорія волосистої частини голови Microsporia of scalp

Гніздова плішивість Nest baldness

Вторинний сифіліс Secondary syphilis

Дискоїдний червоний вовчак Discoid lupus erythematosus

82 / 130
Хворий 45-ти років має скарги на постійний головний біль, біль у серці, колінних суглобах. Три роки тому був укус кліща з кільцевою еритемою на шкірі. Об’єктивно: акродерматит, двобічна пірамідна недостатність, розлади координації. Який діагноз найбільш імовірний? A 45-year-old patient complains of constant headache, pain in the heart, and knee joints. Three years ago, there was a tick bite with annular erythema on the skin. About' objectively: acrodermatitis, bilateral pyramidal insufficiency, coordination disorders. What is the most likely diagnosis?

Нейробореліоз Neuroborreliosis

Розсіяний склероз Multiple sclerosis

Вірусний енцефаліт Viral encephalitis

Ішемічна енцефалопатія Ischemic encephalopathy

Нейросифіліс Neurosyphilis

83 / 130
Чоловік 32-х років звернувся до сімейного лікаря зі скаргами на нав’язливі спогади про автомобільну аварію, яка була 3 місяці тому, порушення нічного сну, тривогу, зниження настрою і апетиту, загальну слабкість і спустошеність. Що найбільш доцільно призначити? A 32-year-old man presented to his family doctor with complaints of intrusive memories of a car accident 3 months ago, nighttime sleep disturbances, anxiety, decreased mood and appetite, general weakness and desolation. What is the most appropriate to prescribe?

Амітриптилін Amitriptyline

Літію карбонат Lithium carbonate

Сертралін Sertraline

Хлорпромазин Chlorpromazine

Карбамазепін Carbamazepine

84 / 130
У жінки 18-ти років, яка перебуває на лікуванні в стаціонарі з приводу системного червоного вовчака, виявлена позитивна реакція мікропреципітацїї на сифіліс. Клінічних проявів хвороби не виявлено. При дообстеженні - ІФА тре- понемний - реакція негативна. Найбільш імовірний діагноз? An 18-year-old woman, who is being treated in a hospital for systemic lupus erythematosus, had a positive microprecipitation reaction to syphilis. No clinical manifestations of the disease were detected. additional examinations - treponemal ELISA - the reaction is negative. The most probable diagnosis?

Сифіліс третинний Tertiary syphilis

Хибнопозитивна реакція на сифіліс False positive reaction to syphilis

Сифіліс прихований Syphilis hidden

Сифіліс первинний Primary syphilis

Сифіліс вторинний Secondary syphilis

85 / 130
17-річний хлопець на уроці фізичного виховання раптово втратив свідомість. На місці була проведена серцево- легенева реанімація. На момент приїзду швидкої свідомість до пацієнта повернулася. При зборі анамнезу стало відомо, що батько і дід дитини проходили обстеження та лікування у кардіолога, проте помирали в молодому віці. При фі- зикальному огляді дитини встановлено наявність систолічного шуму в серці, гучність якого збільшується при зміні положення тіла. При ехокардіографічному дослідженні зареєстровано потовщення стінки лівого шлуночка більше 15 мм. Яке захворювання можна припустити у даного хворого? A 17-year-old boy suddenly lost consciousness during a physical education class. Cardiopulmonary resuscitation was performed on the spot. At the time of the arrival of the ambulance, the patient regained consciousness. During the collection of anamnesis it became known that the child's father and grandfather were examined and treated by a cardiologist, but died at a young age. Physical examination of the child revealed the presence of a systolic heart murmur, the volume of which increases when the body position is changed. Echocardiographic examination revealed thickening of the left ventricular wall more than 15 mm. What disease can be assumed in this patient?

Дилатаційна кардіоміопатія Dilated cardiomyopathy

Вегетативна дисфункція Vegetative dysfunction

Гіпертрофічна кардіоміопатія Hypertrophic cardiomyopathy

Гостра ревматична лихоманка Acute rheumatic fever

Неревматичний кардит Non-rheumatic carditis

86 / 130
На прийомі у сімейного лікаря дитина, яка народилася доношеною, здоровою і дотепер нічим не хворіла. Дитина може гратися іграшками годину і більше. Ходить вздовж меблів при підтримці за одну руку або самостійно. Має мовний запас із 8-12 слів. За проханням обнімає батьків та прагне схвалення, підтвердження свого успіху. Визначте імовірний вік дитини: At the family doctor's appointment, a child who was born full-term, healthy and has not been sick so far. The child can play with toys for an hour or more. He walks along the furniture with support for one hand or independently. Has a vocabulary of 8-12 words. Upon request, hugs parents and seeks approval, confirmation of his success. Determine the probable age of the child:

9 місяців 9 months

7 місяців 7 months

6 місяців 6 months

12 місяців 12 months

8 місяців 8 months

87 / 130
При плановому обстеженні 22-річної вагітної (ЗО тижнів) двічі у сечі виявлена ізольована бактеріурія. Вагітність перебігає без патології. Визначте тактику ведення вагітної: During a routine examination of a 22-year-old pregnant woman (3 weeks), isolated bacteriuria was detected twice in the urine. The pregnancy proceeds without pathology. Determine the tactics of managing the pregnant woman:

Призначення ампіциліну Prescription of ampicillin

Призначення ципрофлоксацину Ciprofloxacin Prescription

Фізіотерапевтичне лікування Physiotherapy treatment

Фітотерапія Phytotherapy

Динамічне спостереження Dynamic Observation

88 / 130
Хворий 35-ти років впродовж 14-ти років працює в ливарному цеху, де концентрація кварцевого пилу в 4 рази перевищує ГДК. Впродовж 4-х років його турбує кашель, задишка при фізичному навантаженні. При аускультації дихання ослаблене. Рентгенографія ОГК: емфізема, дрібноплямисті тіні по всім легеневим полям. Найбільш імовірний діагноз? A 35-year-old patient has been working in a foundry for 14 years, where the concentration of quartz dust is 4 times higher than the MPC. He has been suffering from cough for 4 years , shortness of breath during physical exertion. On auscultation, breathing is weakened. X-ray of OGK: emphysema, fine plume-like shadows in all lung fields. The most likely diagnosis?

Туберкульоз легенів Pulmonary tuberculosis

Азбестоз Asbestosis

Силікоз Silicosis

хозл hozl

Хронічний бронхіт Chronic bronchitis

89 / 130
Чоловік 68-ми років звернувся до лікаря зі скаргами на сильний головний біль, нудоту, відчуття дискомфорту за грудиною. Протягом 2-х років страждає на гіпертонічну хворобу. Свідомість збережена. Ознак неврологічного дефіциту немає. AT- 220/100 мм рт.ст., ЧСС- 92/хв. ЕКГ: ритм синусовий, ознаки гіпертрофії лівого шлуночка. Яке ускладнення розвилося? A 68-year-old man consulted a doctor with complaints of a severe headache, nausea, a feeling of discomfort behind the sternum. He has been suffering from hypertension for 2 years. Consciousness preserved. There are no signs of neurological deficit. AT - 220/100 mm Hg, heart rate - 92/min. ECG: sinus rhythm, signs of left ventricular hypertrophy. What complication developed?

Розшаровуюча аневризма аорти Dissecting aortic aneurysm

Інфаркт міокарда Myocardial infarction

Гостре порушення мозкового кровообігу Acute cerebrovascular accident

Ускладнений гіпертензивний криз Complicated hypertensive crisis

Неускладнений гіпертензивний криз Uncomplicated hypertensive crisis

90 / 130
Хворий 54-х років звернувся до сімейного лікаря зі скаргами на утруднення дихання, слабкість. Останні два тижні турбував біль та набряк правої нижньої кінцівки. Дані симптоми вперше в житті, раніше на обліку у лікаря не перебував. AT- 110/70 мм рт.ст., ЧСС- 96/хв. Який діагностичний метод має вирішальне значення? A 54-year-old patient turned to his family doctor with complaints of difficulty breathing, weakness. For the past two weeks, he has been troubled by pain and swelling of the right lower limb. These symptoms are the first time in his life , had not previously been registered with a doctor. AT - 110/70 mm Hg, heart rate - 96/min. What diagnostic method is crucial?

Мультиспіральна КТ ангіографія Multispiral CT angiography

Рентгенографія органів грудної клітки X-ray of chest organs

Ехокардіографія Echocardiography

Електрокардіографія Electrocardiography

Функція зовнішнього дихання External breathing function

91 / 130
Хворий 25-ти років до психіатричної лікарні надходить вперше. Збуджений, говорить швидко й голосно, активно жестикулює. Без упину жартує, сміється, з обличчя не сходить посмішка. З лікарем розмовляє на ”ти” пропонує йому зіграти в карти, залицяється до медсестер, запрошує їх на побачення. Розповідає, що останні 2 доби не спав, проте зовсім не відчуває втоми. Який препарат доцільно використати? A 25-year-old patient is admitted to a psychiatric hospital for the first time. He is agitated, speaks quickly and loudly, gestures actively. He jokes non-stop, laughs, a smile never leaves his face. He talks to the doctor in 'you', offers him to play cards, flirts with the nurses, invites them on a date. He says that he hasn't slept for the last 2 days, but he doesn't feel tired at all. What drug should I use?

Снодійний Sleepy

Антидепресант Antidepressant

Седативний Sedative

Антипсихотичний Antipsychotic

Ноотропний Nootropic

92 / 130
Чоловіку 42-х років виконано геміти- реоїдектомію з приводу вузлового еути- реоїдного зоба. Призначення якого лікування найбільш імовірно допоможе запобігти рецидиву захворювання? A 42-year-old man underwent a hemithyroidectomy for a nodular euthyroid goiter. Which treatment is most likely to help prevent disease recurrence?

Антиструмін Anticurrent

Радіоактивний йод Radioactive iodine

Мерказоліл Mercazolil

Тиротропін Thyrotropin

L-тироксин L-thyroxine

93 / 130
Під час профогляду у жінки 50-ти років у правій молочній залозі виявлена пухлина діаметром 5 см, щільної консистенції, без чітких контурів. Шкіра над пухлиною має вигляд лимонної кірки. У пахвовій ділянці пальпується лімфатичний вузол. Який найбільш імовірний діагноз? During the examination of a 50-year-old woman in the right mammary gland, a tumor with a diameter of 5 cm, a dense consistency, without clear contours was found. The skin above the tumor has the appearance of a lemon peel . A lymph node is palpated in the armpit. What is the most likely diagnosis?

Мастит Mastitis

Кіста молочної залози Breast gland cyst

Дифузна мастопатія Diffuse mastopathy

Рак молочної залози Breast cancer

Ліпома молочної залози Lipoma of the mammary gland

94 / 130
У пацієнта 47-ми років мітральний стеноз після гострої ревматичної лихоманки. Скаржиться на прогресуючу задишку, кашель, виражену загальну слабкість. На даний момент не може виконувати легку побутову роботу. Найбільш доцільна тактика лікування? A 47-year-old patient has mitral stenosis after acute rheumatic fever. He complains of progressive shortness of breath, cough, severe general weakness. At the moment, he cannot perform light household work . The most appropriate treatment tactic?

Антибактеріальна терапія Antibacterial therapy

Препарати наперстянки Dingular drugs

Призначення діуретиків Prescription of diuretics

Призначення антикоагулянтів Prescription of anticoagulants

Хірургічне лікування Surgical treatment

95 / 130
Пацієнт 20-ти років звернувся в клініку із скаргами на загальну слабкість, підвищення температури тіла, припухлість та болючість суглобів, висипання на гомілках. На рентгенограмі ОГК збільшені внутрішньогрудні лімфатичні вузли. Найбільш імовірний діагноз: A 20-year-old patient came to the clinic with complaints of general weakness, increased body temperature, swelling and pain in the joints, a rash on the lower legs. On an X-ray of the OGK, enlarged intrathoracic lymphatics nodes. The most likely diagnosis:

Саркоїдоз Sarcoid

Туберкульоз легень Pulmonary tuberculosis

Медіастиніт Mediastinitis

Рак легень Lung cancer

Лімфогранулематоз Lymphogranulomatosis

96 / 130
Пацієнт 37-ми років надійшов у не- фрологічне відділення з діагнозом: хронічна хвороба нирок, І ст. - мезангі- опроліферативний гломерулонефрит. AT- 140/90 мм рт.ст. Добова втрата білка із сечею становить - 5,1 г. У крові: загальний білок - 48,2 г/л; сечовина - 5,7 ммоль/л; креатинін - 76,3 мкмоль/л; холестерин - 8,1 ммоль/л. Яку групу препаратів необхідно призначити? A 37-year-old patient was admitted to the nephrology department with a diagnosis of chronic kidney disease, stage I - mesangiproliferative glomerulonephritis. AT- 140/90 mm Hg. The daily loss of protein in the urine is 5.1 g. In the blood: total protein - 48.2 g/l, urea - 5.7 mmol/l, creatinine - 76.3 μmol/l, cholesterol - 8 ,1 mmol/l. What group of drugs should be prescribed?

Антибіотики Antibiotics

Нестероїдні протизапальні Nonsteroidal anti-inflammatory drugs

Ілюкокортикостероїди Ilucocorticosteroids

Сульфаніламідні Sulfonamides

Нітрофурани Nitrofurans

97 / 130
У відділення надійшов хворий 30-ти років зі скаргами на часті рідкі випорожнення з кров’ю і слизом, підвищення температури до 37.7'С, переймоподібний біль у животі. Пальпаторно відзначається болючість по ходу товстої кишки. При ректороманоскопії: стінка кишечника набрякла, легкоранима, ерозії і виразки, значна кількість крові та слизу. Імовірний попередній діагноз: A 30-year-old patient came to the department with complaints of frequent liquid stools with blood and mucus, temperature rise to 37.7°C, spasm-like abdominal pain. On palpation, there is pain along the course of the large intestine. During rectoromanoscopy: the wall of the intestine is swollen, easily wounded, erosions and ulcers, a significant amount of blood and mucus. Presumable preliminary diagnosis:

Неспецифічний виразковий коліт Nonspecific ulcerative colitis

Хвороба Крона Crohn's disease

Целіакія Celiac

Поліпоз товстої кишки Colon polyposis

Дизентерія Dysentery

98 / 130
Хлопчик 7-ми років звернувся по допомогу до педіатра зі скаргами на повторювані епізоди нудоти і блювання протягом останніх 3-х днів. Блювання без домішок крові і жовчі, виникає переважно вранці і повторюється до 10-ти разів на день. Вживає здорову їжу. Це третій епізод блювання за останні 6 місяців. Об’єктивно: запах ацетону з рота, легкий ступінь дегідратації. Інші показники в нормі. Який діагноз найімовірніший? A 7-year-old boy sought help from a pediatrician with complaints of repeated episodes of nausea and vomiting during the past 3 days. Vomiting without blood and bile impurities, occurs mainly in the morning and is repeated up to 10 times a day. Eats healthy food. This is the third episode of vomiting in the last 6 months. Objectively: the smell of acetone from the mouth, a slight degree of dehydration. Other indicators are normal. What is the most likely diagnosis?

Іострий панкреатит Acute pancreatitis

Виразкова хвороба шлунку Gastric ulcer disease

Іострий гастрит Acute gastritis

Хронічний гастрит Chronic gastritis

Синдром циклічного блювання Syndrome of cyclic vomiting

99 / 130
У дитини 9-ти років з’явився біль у верхній третині правої гомілки, підвищилась температура тіла до 39°С. Дитина не може стати на праву ногу. З анамнезу відомо, що дитина мала травму гомілки та перенесла ангіну. Яке захворювання можна припустити? A 9-year-old child developed pain in the upper third of the right shin, the body temperature rose to 39°C. The child cannot stand on his right leg. in the anamnesis, it is known that the child had a leg injury and suffered angina. What disease can be assumed?

Іострий ревматизм Acute rheumatism

Туберкульозний артрит Tuberculous arthritis

Іострий гематогенний остеомієліт Acute hematogenous osteomyelitis

Перелом кісток правої гомілки Fracture of the bones of the right tibia

Злоякісна пухлина кістки Malignant bone tumor

100 / 130
Підліток 15-ти років напередодні був у контакті із хворим на кір. При огляді - здоровий. Не щеплений за переконаннями батьків. Які заходи рекомендовані для профілактики виникнення захворювання? A 15-year-old teenager was in contact with a measles patient the day before. On examination, he is healthy. He has not been vaccinated according to his parents' beliefs. What measures are recommended to prevent the occurrence of the disease?

Вакцинація проти кору протягом перших 72 годин після контакту з хворим Measles vaccination within the first 72 hours after contact with the patient

Вакцинація проти кору через 4 доби після контакту з хворим Measles vaccination 4 days after contact with the patient

Спостереження протягом 5 діб, при появі проявів захворювання - вакцинація проти кору Observation for 5 days, when symptoms of the disease appear - vaccination against measles

Ніякі None

Антибіотики широкого спектру дії Broad-spectrum antibiotics

101 / 130
Жінка 42-х років звернулася до лікаря зі скаргами на м’язову слабкість у верхніх і нижніх кінцівках. Об’єктивно: у періорбітальних ділянках еритема з бузковим відтінком, макульозна еритема на розгинальних поверхнях пальців. В аналізі крові виявлено суттєве підвищення рівня креатинфосфокінази. Яким є імовірний діагноз? A 42-year-old woman consulted a doctor with complaints of muscle weakness in the upper and lower limbs. Objectively: in the periorbital areas erythema with a lilac tint, macular erythema on the extensor surfaces of the fingers. A blood test revealed a significant increase in the level of creatine phosphokinase. What is the probable diagnosis?

Дерматоміозит Dermatomyositis

Хвороба Кушинга Cushing's disease

Системний червоний вовчак Systemic lupus erythematosus

Системна склеродермія Systemic scleroderma

Гіпопаратиреоз Hypoparathyroidism

102 / 130
Хворий на гемофілію А 18-ти років звернувся із скаргами на гострий біль у лівому колінному суглобі та збільшення його в об’ємі. Суглоб різко болючий під час рухів та пальпації, шкіра над ним гаряча на дотик. Яке лікування слід застосувати? An 18-year-old hemophiliac A patient complained of sharp pain in the left knee joint and an increase in its volume. The joint is sharply painful during movements and palpation, the skin over it is hot to the touch. What treatment should be applied?

В/в введення концентрату чинника IX IV administration of factor IX concentrate

Накласти холод та іммобілізувати суглоб Apply cold and immobilize the joint

Провести лікувальну пункцію суглоба Perform medical puncture of the joint

В/в введення концентрату чинника VIII IV administration of factor VIII concentrate

В/в введення свіжозамороженої плазми IV administration of fresh frozen plasma

103 / 130
Новонароджена доношена дитина з масою тіла 4500 г народилася в асфіксії з оцінкою за шкалою Апгар 4-6 балів. В пологах утруднене виведення плечового поясу. У неврологічному статусі: загальномозкові розлади, виявлений тотальний верхній млявий парез - ручка атонічна, пронована, не викликаються рефлекси - хапальний, Бабкіна. Вкажіть рівень ураження спинного мозку: A newborn full-term child with a body weight of 4500 g was born asphyxiated with an Apgar score of 4-6 points. During delivery, the removal of the shoulder girdle was difficult. In the neurological status: cerebral disorders, total upper flaccid paresis was detected - the handle is atonic, pronated, reflexes are not evoked - grasping, Babkin. Specify the level of damage to the spinal cord:

Шийні сегменти СІП-С IV Cervical segments of SIP-C IV

Грудні сегменти TVI-TVII Thoracic segments TVI-TVII

Шийно-грудні сегменти CV-TI Cervothoracic segments CV-TI

Шийні сегменти СІ-СП Cervical segments of SI-SP

Грудні сегменти TI-TV TI-TV Chest Segments

104 / 130
Дитина 3-х місяців доставлена в лікарню зі скаргами на мляве смоктання та появу задишки, яка посилюється під час годування, неспокій. Симптоми з’явилися протягом останніх годин. Шкіра бліда, з попелястим відтінком, кінцівки холодні. Аксилярна температура - 36,8°С*. Частота дихання - 66/хв. Частоту серцевих скорочень підрахувати не вдається. Пульс слабкий. Печінка пальпується на 4 см нижче реберної дуги. На ЕКГ: ЧСС - 230/хв., ритм правильний, зубець Р на Т, комплекси тривалістю 0,08с. Який препарат вибору для надання невідкладної допомоги даній дитині? A 3-month-old baby was brought to the hospital with complaints of sluggish sucking and shortness of breath, which worsens during feeding, restlessness. Symptoms appeared during the last hours. The skin is pale, with an ashy tint, the extremities are cold. Axillary temperature - 36.8°С*. Respiration rate - 66/min. It is not possible to calculate the heart rate. The pulse is weak. The liver is palpable 4 cm below the costal arch. On the ECG: HR - 230/min., rhythm is correct, wave P on T, complexes lasting 0.08s. What is the drug of choice for providing emergency care to this child?

Пропранолол Propranolol

Аденозин Adenosine

Верапаміл Verapamil

Дигоксин Digoxin

Лідокаїн Lidocaine

105 / 130
До сімейного лікаря звернувся пацієнт з астено-вегетативним синдромом. Пацієнт пред’являв скарги на порушення сну, частий головний біль, запаморочення, дратівливість, підвищену втомлюваність, порушення пам’яті. Пацієнт протягом 2-х років не працював, проживає в житловому будинку, який розташований поруч з аеропортом. З впливом якого чинника найімовірніше можливе виникнення даних симптомів? A patient with astheno-vegetative syndrome consulted a family doctor. The patient complained of sleep disturbances, frequent headaches, dizziness, irritability, increased fatigue, memory impairment 'yati. The patient has not worked for 2 years, lives in a residential building located near the airport. What factor is most likely to cause these symptoms?

Хімічне забруднення атмосферного повітря Chemical air pollution

Вібрація Vibration

Ультразвук Ultrasound

Шум Noise

Електромагнітні поля Electromagnetic fields

106 / 130
Новонароджена дитина народилася на 37 тижні вагітності. Дитина млява, шкіра іктерична, на шкірі тулуба незначний геморагічний висип, рефлекси пригнічені, виявлений ністагм та хоріорети- ніт. Іепатоспленомегалія. В ПЛР крові виявлено ДНК цитомегаловірусу. Який етіотропний препарат є найбільш ефективним? A newborn baby was born at the 37th week of pregnancy. The baby is lethargic, the skin is icteric, there is a slight hemorrhagic rash on the skin of the body, reflexes are depressed, nystagmus and chorioretinitis are detected. Hepatosplenomegaly. Cytomegalovirus DNA was detected in PCR of blood. Which etiotropic drug is the most effective?

Ацикловір Acyclovir

Іанцикловір Ianciclovir

Рибавірин Ribavirin

Ремантадин Remantadine

Озельтамівір Oseltamivir

107 / 130
До дерматолога звернувся чоловік віком 43 роки у зв’язку із наявністю висипки на шкірі кистей, яка супроводжується свербежем. Хворіє 2 тижні. Об’єктивно: у вказаних місцях відзначається гіперемія та набряк шкіри, везикули, мо- кнуття, екскоріації на місцях розчухування. Який найбільш імовірний діагноз у хворого? A 43-year-old man consulted a dermatologist due to the presence of a rash on the skin of the hands, which is accompanied by itching. He has been sick for 2 weeks. Objectively: in the indicated places there is hyperemia and swelling of the skin, vesicles, wetting, excoriations at the places of scratching. What is the most likely diagnosis for the patient?

Дерматит контактний Contact dermatitis

Дерматит алергічний Allergic dermatitis

Токсикодермія Toxicoderma

Екзема мікробна Microbial eczema

Екзема справжня Eczema is true

108 / 130
Завідувач відділення стаціонару хоче провести експертну оцінку виконання лікарями-ординаторами медико- технологічних стандартів обслуговування пацієнтів. Яку документацію він повинен перевірити з цією метою? The head of the hospital's department wants to conduct an expert assessment of the fulfillment of medical and technological standards of patient care by resident doctors. What documentation should he check for this purpose?

Статистичні карти пацієнтів, що вибули із стаціонару Statistical charts of patients discharged from the hospital

Річний звіт лікувально- профілактичного закладу Annual report of the medical and preventive institution

Медичні карти стаціонарних хворих Medical charts of inpatients

Журнал обліку оперативних втручань Operational interventions accounting log

Карти лікарських призначень Maps of medical appointments

109 / 130
Місто забезпечується питною водою з підземних артезіанських джерел, яка характеризується високою якістю і чистотою. Який з перерахованих методів обробки води, отриманої з підземних міжпластових напірних джерел, передбачається санітарними вимогами? The city is provided with drinking water from underground artesian springs, which is characterized by high quality and purity. Which of the listed methods of treatment of water obtained from underground interlayer pressure springs is provided by sanitary requirements ?

Коагуляція Coagulation

Знезараження Disinfection

Опріснення Desalination

Знебарвлення Fading

Дефторування Defluorination

110 / 130
У чоловіка 55-ти років 2 роки тому верифіковано цироз печінки. Впродовж 2-х тижнів стан погіршився, з’явились сонливість, запаморочення, втрата орієнтації у просторі і часі. Напередодні відзначав день народження в ресторані. Який синдром зумовив погіршення стану хворого? A 55-year-old man was diagnosed with liver cirrhosis 2 years ago. Over the course of 2 weeks, his condition worsened, drowsiness, dizziness, loss of orientation in space and times. He celebrated his birthday in a restaurant the day before. What syndrome caused the patient's condition to deteriorate?

Холестазу Cholestasis

Печінкової енцефалопатії Hepatic encephalopathy

Жовтяниці Jaundice

Гіперспленізму Hypersplenism

Цитолізу Cytolysis

111 / 130
Підліток госпіталізований зі скаргами на обмеженість рухів у нижніх кінцівках. Захворювання розпочалось 2 дні тому з підвищення температури тіла до 38,3°С*, рідких випорожнень 3-4 рази на добу. Об’єктивно: температура тіла - 36,8°С*, активні рухи в нижніх кінцівках відсутні, у зоні ураження - арефлексія, гіпотонія м’язів, чутливість збережена. Менінгеальні симптоми слабко позитивні. Яке захворювання можна припусти-ти? A teenager was hospitalized with complaints of limited movement in the lower limbs. The disease began 2 days ago with an increase in body temperature to 38.3°C*, liquid stools 3-4 times a day. Objectively: body temperature - 36.8°С*, active movements in the lower limbs are absent, in the affected area - areflexia, muscle hypotonia, sensitivity is preserved. Meningeal symptoms are weakly positive. What disease can be assumed ?

Іерпетичний менінгоенцефаліт Herpetic meningoencephalitis

Субарахноїдальний крововилив Subarachnoid hemorrhage

Травма поперекового відділу хребта Injury of the lumbar spine

Поліомієліт Polio

Ботулізм Botulism

112 / 130
Після перенесеного геморагічного інсульту хворий з метою відновлення мови та рухів лівої нижньої та верхньої кінцівок проходить курс масажу, фізпро- цедур та занять з логопедом. До якого виду профілактики належать вказані заходи реабілітації? After suffering a hemorrhagic stroke, the patient undergoes a course of massage, physical procedures, and classes with a speech therapist in order to restore speech and movements of the left lower and upper extremities. What type of prevention does this include indicated rehabilitation measures?

Громадська профілактика Public prevention

Первинна профілактика Primary prevention

Індивідуальна профілактика Individual prevention

Вторинна профілактика Secondary prevention

Третинна профілактика Tertiary prevention

113 / 130
Жінка 23-х років. Скарги на ріжучий біль внизу живота, почастішання сечовипускання, підвищення температури тіла до У1,7°С, збільшення виділень зі статевих шляхів жовтуватого кольору. Остання менструація 18 днів назад. В анамнезі: погіршення стану через 5-6 днів після незахищеного статевого акту. Симптом Щоткіна негативний. Біману- ально: двобічне збільшення додатків. Діагноз: 23-year-old woman. Complaints of cutting pain in the lower abdomen, increased frequency of urination, increase in body temperature up to В1.7°C, increased secretions from the genital tract of a yellowish color . The last menstruation was 18 days ago. In the anamnesis: deterioration of the condition 5-6 days after unprotected sexual intercourse. Shotkin's symptom is negative. Bimanual: bilateral enlargement of the appendages. Diagnosis:

Бактеріальний вагіноз Bacterial vaginosis

Двобічний сальпінгоофорит Bilateral salpingo-oophoritis

Кандидозний вульвовагініт Candida vulvovaginitis

Апендицит Appendicitis

Ендометрит Endometritis

114 / 130
32-річна жінка скаржиться на нерегулярні менструації з затримками до 2-3 місяців, значну прибавку маси тіла, гірсутизм. Заміжня 5 років. Вагітностей не було. При піхвовому дослідженні матка незначно зменшена, з обох сторін визначаються щільні, рухомі яєчники до 4-5 см в діаметрі. Яку патологію можна припустити в даному випадку? A 32-year-old woman complains of irregular menstruation with delays of up to 2-3 months, a significant increase in body weight, hirsutism. She has been married for 5 years. There have been no pregnancies. With vaginal on examination, the uterus is slightly reduced, dense, mobile ovaries up to 4-5 cm in diameter are detected on both sides. What pathology can be assumed in this case?

Двобічні кісти яєчників Bilateral ovarian cysts

Туберкульоз придатків матки Tuberculosis of uterine appendages

Хронічний двобічний сальпінгіт Chronic bilateral salpingitis

Синдром полікістозних яєчників Polycystic ovary syndrome

Гіпоменструальний синдром Hypomenstrual syndrome

115 / 130
У дитини 4,5 років з вираженими ознаками зневоднення свідомість на рівні сопору, загальний ціаноз, токсичне дихання, анурія, AT- 60/20 ммрт.ст., ембріо- кардія, відсутність пульсу на променевій артерії. Який болюс розчину NaCl 0,9% необхідно ввести на першому етапі невідкладної допомоги? A 4.5-year-old child with pronounced signs of dehydration has consciousness at the level of sopor, general cyanosis, toxic breathing, anuria, AT- 60/20 mmHg, embryocardia, absence of a pulse on the radial artery. What bolus of 0.9% NaCl solution should be administered at the first stage of emergency care?

80 мл/кг 80 ml/kg

100 мл/кг 100 ml/kg

ЗО мл/кг 3 ml/kg

50 мл/кг 50 ml/kg

60 мл/кг 60 ml/kg

116 / 130
Чоловік 27-ми років 3 роки тому переніс важку черепно-мозкову травму. Останній рік у нього з’явилися стани, коли він під час будь-якої роботи раптово застигає, не втрачаючи м’язового тонусу, вираз обличчя стає маскоподібним, що триває 1-2 хвилини. Після цього він повертається до попередньої діяльності, але подібні ”відключення” не пам’ятає. Препарати якої групи доцільно призначити? A 27-year-old man suffered a severe brain injury 3 years ago. In the last year, he has had conditions in which, during any work, he suddenly stiffens, without losing muscle tone, the facial expression becomes mask-like, which lasts 1-2 minutes. After that, he returns to his previous activity, but he does not remember such 'disconnections'. What group of drugs should be prescribed?

Ноотропні засоби Nootropics

Антидепресанти Antidepressants

Антигіпертензивні засоби Antihypertensive drugs

Антипсихотики Antipsychotics

Антиконвульсанти Anticonvulsants

117 / 130
Жінка 78-ми років, яка страждає на артеріальну гіпертензію, мала три епізоди раптової транзиторної втрати зору на ліве око. При аускультації сонних артерій в точках біфуркації з обох боків вислуховується шум. Яке додаткове дослідження найбільш доцільно призначи-ти? A 78-year-old woman suffering from arterial hypertension had three episodes of sudden transient loss of vision in the left eye. On auscultation of the carotid arteries at the bifurcation points on both sides noise is heard. What additional research is most appropriate to prescribe?

Люмбальна пункція Lumbar puncture

Нейросонографія Neurosonography

МРТ головного мозку MRI brain

КТ головного мозку CT brain

Дуплексне сканування судин шиї Duplex scan of neck vessels

118 / 130
Хвора 35-ти років скаржиться на перебої в роботі серця, зниження ваги, біль передньої поверхні шиї, що почалися після ГРВІ. Пульс - 110/хв., AT-130/60 мм рт.ст. Щитоподібна залоза незначно збільшена, болюча при пальпації. ШОЕ- 40 мм/год. ТТГ- 0,05 мОд/л, вТ4 - 34,3 пмоль/л. Який найбільш імовірний діагноз? A 35-year-old patient complains of heart failure, weight loss, pain in the front surface of the neck, which began after SARS. Pulse - 110/min., AT -130/60 mm Hg. The thyroid gland is slightly enlarged, painful on palpation. ESR - 40 mm/h. TSH - 0.05 mU/l, vT4 - 34.3 pmol/l. What is the most likely diagnosis?

Тиреоїдит Хашимото Hashimoto's thyroiditis

Підгострий тиреоїдит Subacute thyroiditis

Гострий тиреоїдит Acute thyroiditis

Дифузний токсичний зоб Diffuse toxic goiter

Пароксизмальна тахікардія Paroxysmal tachycardia

119 / 130
Жінка 46-ти років знайшла при пальпації вузол в ділянці щитоподібної залози розміром 2-3 см, який зміщується при ковтанні, з чіткими межами. При скануванні визначається ”холодний вузол” у правій частці щитоподібної залози. Лімфовузли шиї не збільшені. При пункції отримано клітини проліферую- чого епітелію. Яка лікувальна тактика? During palpation, a 46-year-old woman found a 2-3 cm nodule in the area of the thyroid gland, which shifts when swallowing, with clear boundaries. During the scan, a 'cold 'node' in the right lobe of the thyroid gland. The lymph nodes of the neck are not enlarged. Proliferating epithelial cells were obtained during the puncture. What are the treatment tactics?

Спостереження Observations

Іеміструмектомія Iemistrumectomy

Правобічна гемітиреоїдектомія з терміновим гістологічним дослідженням Right-sided hemithyroidectomy with urgent histological examination

Променеве лікування Radiation treatment

Тиреоїдектомія з висіченням клітковини шиї за Крайнем Thyroidectomy with neck tissue excision according to Kraine

120 / 130
Хворий 30-ти років надійшов в клініку зі скаргами на задишку, гарячку, біль за грудиною, який посилюється при закиданні голови назад. З дні тому хворому проведена екстракція сьомого зуба нижньої щелепи праворуч і розкрита флегмона дна порожнини рота. Незважаючи на це, стан хворого прогресивно погіршувався. При рентгенологічному обстеженні в клініці виявлено розширення тіні середостіння і зниження прозорості його відділів. Яке захворювання виникло у даного хворого? A 30-year-old patient came to the clinic with complaints of shortness of breath, fever, pain behind the sternum, which worsens when the head is thrown back. The patient underwent an extraction of the seventh a tooth of the lower jaw on the right and an open phlegmon of the floor of the oral cavity. Despite this, the patient's condition progressively worsened. An X-ray examination in the clinic revealed an expansion of the mediastinum shadow and a decrease in the transparency of its parts. What disease did this patient have?

Заглотковий абсцес Pharyngeal abscess

інфекційний ендокардит infective endocarditis

Ексудативний перикардит Exudative pericarditis

Гнійний тиреоїдит Suppurative thyroiditis

Гострий гнійний медіастиніт Acute purulent mediastinitis

121 / 130
Хворому 65-ти років встановлено діагноз рак головного мозку TV стадії. Чоловік з вираженими епілептичними нападами та больовим синдромом. Який вид медичної допомоги необхідний для полегшення стану хворого? A 65-year-old patient has been diagnosed with stage TV brain cancer. A man with severe epileptic seizures and pain syndrome. What type of medical care is needed to alleviate the patient's condition?

Вторинна медична допомога Secondary medical care

Паліативна медична допомога Palliative medical care

Санаторно-курортна допомога Sanatorium and spa assistance

Первинна медична допомога Primary medical care

Третинна медична допомога Tertiary medical care

122 / 130
Лікар медичного кабінету загальноосвітньої школи повинен провести оцінку фізичного розвитку дітей та підлітків. Ця оцінка проводиться на підставі зіставлення індивідуальних даних з: The doctor of the medical office of the secondary school must conduct an assessment of the physical development of children and adolescents. This assessment is carried out on the basis of comparing individual data with:

Загальносвітовими стандартами фізичного розвитку Global standards of physical development

Даними наукових досліджень в періодичних фахових виданнях Data of scientific research in periodical specialist publications

Дані не порівнюють Data does not compare

Регіональними стандартами фізичного розвитку By regional standards of physical development

Даними засобів масової інформації Media data

123 / 130
При проведенні поточного профілактичного медичного огляду робітника цеху з виробництва автомобільних акумуляторів встановлено: шкіра бліда з сірувато-землистим відтінком, видимі слизові бліді. На яснах, переважно біля передніх зубів, є зміна кольору слизової у бузковий колір у вигляді смуги. Є тремор пальців рук. При пальпації рук є біль по ходу нервів. Назвіть токсичну сполуку та основний шлях її надходження до організму людини в умовах виробництва: During the current preventive medical examination of a worker in the workshop for the production of automobile batteries, it was established: the skin is pale with a grayish-earthy hue, the mucous membranes are pale. On the gums, mainly near the front teeth , there is a change in the color of the mucous membrane to a lilac color in the form of a band. There is a tremor of the fingers. When palpating the hands, there is pain along the course of the nerves. Name the toxic compound and the main route of its entry into the human body under production conditions:

Кадмій, перкутанний Cadmium, percutaneous

Арсен, інгаляційний Arsenic, inhalation

Нікель, перкутанний Nickel, Percutaneous

Свинець, інгаляційний Lead, inhalation

Ртуть, пероральний Mercury, oral

124 / 130
Хвора 27-ми років надійшла до гінекологічного відділення зі скаргами на різкий біль в ділянці зовнішніх статевих органів, що посилюється при ходьбі, підвищення температури тіла до 39,2°С*, озноб. Захворіла тиждень тому, після переохолодження. Остання менструація З тижні тому. При огляді зовнішніх статевих органів в ділянці великої статевої губи визначається болюче пухлиноподібне утворення розмірами 3x4 см, шкіра над ним гіперемована, гаряча на дотик, при пальпації відмічається флуктуація. Діагноз: A 27-year-old patient came to the gynecology department with complaints of sharp pain in the area of the external genitalia, which worsens when walking, an increase in body temperature to 39.2° C*, chills. Fell sick a week ago, after hypothermia. Last menstruation Since a week ago. During examination of the external genitalia in the area of the labia majora, a painful tumor-like formation measuring 3x4 cm is determined, the skin above it is hyperemic, hot to the touch, fluctuation is noted during palpation. Diagnosis:

Рак вульви Vulvar cancer

Фурункул великої статевої губи Furnish of labia majora

Абсцес бартолінієвої залози Bartholin gland abscess

Кіста бартолінієвої залози Bartholin gland cyst

Ліпома зовнішніх статевих органів Lipoma of the external genitalia

125 / 130
Пацієнтка 55-ти років звернулася до лікаря жіночої консультації зі скаргами на біль внизу живота, появу кров’яних виділень через 3 роки після припинення менструації. Які додаткові методи обстеження необхідно провести у даної пацієнтки для уточнення діагнозу? A 55-year-old patient turned to the doctor of the women's consultation with complaints of pain in the lower abdomen, the appearance of bloody discharge 3 years after the cessation of menstruation. What are the additional methods of examination is it necessary to conduct this patient to clarify the diagnosis?

Лапароскопія Laparoscopy

Роздільне діагностичне вишкрібання матки Separate diagnostic curettage of the uterus

Кольпоскопія Colposcopy

УЗД органів малого таза Pelvic ultrasound

Зішкріб на онкоцитологію Scraping for oncocytology

126 / 130
Хлопчик 10-ти років скаржиться на підвищення температури тіла до 38,5°С*, припухлість суглобів пальців рук та ніг, ранкову скутість, біль у шийному відділі хребта. Проведено обстеження. У крові: еритроцити - 2,6 • 1012/л, НЬ- 85 г/л, лейкоцити -16,5 • 109/л, є,- 2%, п,- 8%, с,- 68%, л,- 16%, м,- 6%, швидкість осідання еритроцитів - 28 мм/год. Позитивний ревматоїдний фактор. Диспротеїнемія з гіперглобулінемією. Встановіть діагноз: A 10-year-old boy complains of an increase in body temperature up to 38.5°C*, swelling of the joints of the fingers and toes, morning stiffness, pain in the cervical spine An examination was carried out. In the blood: erythrocytes - 2.6 • 1012/l, Hb - 85 g/l, leukocytes -16.5 • 109/l, yes, - 2%, p, - 8%, c, - 68 %, l, - 16%, m, - 6%, erythrocyte sedimentation rate - 28 mm/h. Positive rheumatoid factor. Dysproteinemia with hyperglobulinemia. Establish a diagnosis:

Іостра ревматична лихоманка Acute rheumatic fever

Реактивний артрит Reactive arthritis

Ювенільний ревматоїдний артрит Juvenile rheumatoid arthritis

Остеоартрит Osteoarthritis

Токсичний синовіїт Toxic synovitis

127 / 130
У 10-річної дівчинки на тлі підвищеної до 37,5ОС* температури тіла спостерігаються помірне збільшення лімфовузлів шиї, сірувато-білі нашарування на мигдаликах у вигляді щільної плівки. Плівка важко знімається, поверхня під нею кровоточить. Набряку шиї немає. Попередній діагноз: A 10-year-old girl has a moderate increase in neck lymph nodes, grayish-white layering on the tonsils in the form of a dense film against the background of a body temperature increased to 37.5 ОС*. Film difficult to remove, the surface beneath it bleeds. There is no swelling of the neck. Preliminary diagnosis:

Інфекційний мононуклеоз Infectious mononucleosis

Стерптококовий тонзилофарингіт Streptococcal tonsillopharyngitis

Скарлатина Scarlatina

Дифтерія мигдаликів Diphtheria of tonsils

Ангіна Симановського-Венсана Symanovsky-Vincent angina

128 / 130
Доношений новонароджений вагою 3900 г. За даними УЗД: вроджена вада серця, коарктація аорти без гіпоплазії дуги аорти, м’язовий дефект міжшлуно- чкової перегородки діаметром 2 мм. Яка тактика лікування? A full-term newborn weighing 3900 g. According to ultrasound: congenital heart disease, coarctation of the aorta without hypoplasia of the aortic arch, muscle defect of the interventricular septum with a diameter of 2 mm. What is the treatment strategy?

Ушивання дефекту міжшлуночкової перегородки Suture of ventricular septal defect

Резекція коарктації аорти Resection of coarctation of the aorta

Пластика дефекту міжшлуночкової перегородки Ventricular septal defect plastic

Пластика дуги аорти Aortic arch plastic

Пластика дефекту міжшлуночкової перегородки та резекція коарктації аорти Ventricular septal defect repair and resection of coarctation of the aorta

129 / 130
У хворого 48-ми років під час профілактичного огляду на рентгенографії ОГК у паренхімі верхньої частки правої легені виявлена промениста тінь до 7 см у діаметрі, яка прилягає до грудної стінки. Який діагностичний метод найбільш інформативний? In a 48-year-old patient, during a preventive X-ray examination, a radiant shadow up to 7 cm in diameter was found in the parenchyma of the upper lobe of the right lung, which is adjacent to the chest wall . Which diagnostic method is the most informative?

Фібробронхоскопія Fibrobronchoscopy

Трансторакальна пункція Transthoracic puncture

Визначення онкомаркерів Definition of tumor markers

Діагностична торакотомія Diagnostic thoracotomy

Аналіз мокроти на клітини злоякісного новоутворення Analysis of sputum for malignant neoplasm cells

130 / 130
Хворий 52-х років скаржився на раптову слабкість і оніміння в лівих кінцівках, утруднення при ходьбі. Об’єктивно відзначалися лівобічні гемігіпестезія і легкий геміпарез. Через 4 години стан хворого нормалізувався, вогнищева симптоматика регресувала, хворий зміг нормально ходити. AT-120/80 мм рт.ст. Який найбільш імовірний діагноз? A 52-year-old patient complained of sudden weakness and numbness in the left limbs, difficulty walking. Objectively, left-sided hemipaesthesia and mild hemiparesis were noted. After 4 hours, the condition the patient recovered, focal symptoms regressed, the patient was able to walk normally. AT-120/80 mm Hg. What is the most likely diagnosis?

Гіпертензивний криз Hypertensive crisis

Іеморагічний інсульт Hemorrhagic stroke

Ішемічний інсульт Ischemic stroke

Транзиторна ішемічна атака Transient ischemic attack

Асоційована мігрень Associated Migraine